Bonus study guide - Exam 3 - NURS 103

Lakukan tugas rumah & ujian kamu dengan baik sekarang menggunakan Quizwiz!

around the mouth and the fingernail beds

A nurse is monitoring a patient with a respiratory problem for the presence of cyanosis. Which are the most appropriate sites to assess?

ANS: C Because the purpose of β-blocker therapy for patients with esophageal varices is to decrease the risk for bleeding. Although propranolol is used to treat hypertension, angina, and tachycardia, the purpose for use in this patient is to decrease the risk for bleeding from esophageal varices.

25. A 49-year-old female patient with cirrhosis and esophageal varices has a new prescription for propranolol (Inderal). Which finding is the best indicator that the medication has been effective? b. Blood pressure is 140/90 mm Hg. c. Stools test negative for occult blood.

Before testing, the nurse would assess the patients mental status and ability to follow directions.

25. During the taking of the health history of a 78-year-old man, his wife states that he occasionally has problems with short-term memory loss and confusion: He cant even remember how to button his shirt. When assessing his sensory system, which action by the nurse is most appropriate?

ANS: C Esophageal varices are dilated submucosal veins. The therapeutic action of H2-receptor blockers in patients with esophageal varices is to prevent irritation and bleeding from the varices caused by reflux of acid gastric contents.

26. Which response by the nurse best explains the purpose of ranitidine (Zantac) for a patient admitted with bleeding esophageal varices? a. The medication will reduce the risk for aspiration. c. The medication will prevent irritation of the enlarged veins.

ANS: D Some medications can increase the risk for NAFLD, and they should be eliminated. NAFLD is not associated with hepatitis, weight loss is usually indicated, and variceal bleeding would not be a concern in a patient with asymptomatic NAFLD.

30. Which action will be included in the care for a patient who has recently been diagnosed with asymptomatic nonalcoholic fatty liver disease (NAFLD)? b. Draw blood for hepatitis serology testing. d. Review the patient's current medication list.

ANS: B Pegylated α-interferon is administered weekly. The other medications are appropriate for a patient with chronic hepatitis C infection.

31. A 34-year old patient with chronic hepatitis C infection has several medications prescribed. Which medication requires further discussion with the health care provider before administration? a. Ribavirin (Rebetol, Copegus) 600 mg PO bid b. Pegylated α-interferon (PEG-Intron, Pegasys) SQ daily

ANS: C This patient's history and fever suggest possible spontaneous bacterial peritonitis, which would require rapid assessment and interventions such as antibiotic therapy.

32. During change-of-shift report, the nurse learns about the following four patients. Which patient requires assessment first? a. 40-year-old with chronic pancreatitis who has gnawing abdominal pain c. 55-year-old with cirrhosis and ascites who has an oral temperature of 102° F (38.8° C)

ANS: B The highest priority outcome is to maintain nutrition because adequate nutrition is needed for hepatocyte regeneration. Finding a home for the patient and identifying the source of the infection would be appropriate activities, but they do not have as high a priority

33. Which goal has the highest priority in the plan of care for a 26-year-old homeless patient admitted with viral hepatitis who has severe anorexia and fatigue? b. Maintain adequate nutrition. c. Establish a stable environment. d. Identify sources of hepatitis exposure.

ANS: C The nurse's first action should be to determine the patient's hemodynamic status by assessing vital signs. Drawing blood for coagulation studies and inserting an IV catheter are also appropriate. However, the vital signs may indicate the need for more urgent actions.

34. Which action should the nurse in the emergency department take first for a new patient who is vomiting blood? a. Insert a large-gauge IV catheter. b. Draw blood for coagulation studies. c. Check blood pressure (BP), heart rate, and respirations. d. Place the patient in the supine position.

ANS: A Respiratory failure can occur as a complication of acute pancreatitis, and maintenance of adequate respiratory function is the priority goal. The other outcomes would also be appropriate for the patient.

35. The nurse is planning care for a 48-year-old woman with acute severe pancreatitis. The highest priority patient outcome is a. maintaining normal respiratory function. d. having adequate fluid and electrolyte balance.

ANS: C Effective pain management will be necessary in order for the patient to improve nutrition, be receptive to teaching, or manage anxiety or depression.

36. The nurse is caring for a 36-year-old patient with pancreatic cancer. Which nursing action is the highest priority? b. Offer high-calorie, high-protein dietary choices. c. Administer prescribed opioids to relieve pain as needed.

ANS: D Postoperative nursing care after a cholecystectomy focuses on prevention of respiratory complications because the surgical incision is high in the abdomen and impairs coughing and deep breathing.

38. A 51-year-old woman had an incisional cholecystectomy 6 hours ago. The nurse will place the highest priority on assisting the patient to a. perform leg exercises hourly while awake. b. ambulate the evening of the operative day. c. turn, cough, and deep breathe every 2 hours. d. choose preferred low-fat foods from the menu.

Potassium: 2.9 mEq/L (2.9 mmol/l) The potassium level is critically low and can affect cardiac and respiratory status. The nurse should communicate this laboratory value immediately. The creatinine is at the high end of normal, the hemoglobin is normal, and the sodium is only slightly low so these values do not need to be reported immediately.

A preoperative nurse is reviewing morning laboratory values on four patients waiting for surgery. Which result warrants immediate communication with the surgical team?

A nurse assesses a client who has cholecystitis. Which clinical manifestation indicates that the condition is chronic rather than acute? a. Temperature of 100.1° F (37.8° C) b. Positive Murphy's sign c. Light-colored stools d. Upper abdominal pain after eating

ANS: C Jaundice, clay-colored stools, and dark urine are more commonly seen with chronic cholecystitis. The other symptoms are seen equally with both chronic and acute cholecystitis.

Assess the pedal pulses

An emergency department nurse cares for a client who sustained a crush injury to the right lower leg. The client reports numbness and tingling in the affected leg. Which action should the nurse take first?

Tachycardia Confusion Symptoms of hemorrhage and hypovolemic shock include hypotension, tachycardia, tachypnea, pallor, diaphoresis, cool and clammy skin, and confusion.

An emergency room nurse assesses a client with potential liver trauma. Which clinical manifestations should alert the nurse to internal bleeding and hypovolemic shock? (Select all that apply.)

Fatigue despite adequate rest Shortness of breath Indigestion

An emergency room nurse assesses a female patient. Which assessment findings would alert the nurse to request a prescription for an electrocardiogram? (Select all that apply.)

anemia, infection, friction and shearing forces against the skin

Bedsides pressure, what other factors contribute to the development of decubitus ulcers?

Hepatitis A Hepatitis A is spread person to person, by the fecal-oral route, and through contaminated food or water. Good hand washing is critical in preventing its spread. The virus can survive on contaminated objects for weeks. Irritable bowel syndrome is the result of increased intestinal motility and is not contagious. Ulcerative colitis and cirrhosis are not infectious.

Careful hand washing before and after contact can prevent the spread of which condition in day care and school settings?

Vertigo

During the taking of the health history, a patient tells the nurse that it feels like the room is spinning around me. The nurse would document this finding as:

Ligaments

Fibrous bands running directly from one bone to another that strengthen the joint and help prevent movement in undesirable directions are called:

blood urea nitrogen (BUN) level. The decreased urine output suggests decreased renal perfusion, and monitoring of renal function is needed. There is no indication that infection is a concern, so antibiotic therapy and a WBC count are not needed. The IV rate may be increased because hypovolemia may be contributing to the patient's decreased urinary output.

Several hours after an open surgical repair of an abdominal aortic aneurysm, the UAP reports to the nurse that urinary output for the past 2 hours has been 40 mL. The nurse notifies the health care provider and anticipates an order for a(n)

Hypertonicity and poor control of posture, balance, and coordinated motion.

Spastic cerebral palsy is characterized by

Infuse lactated Ringer's solution at 200 mL/hr. Correct: The client's most immediate concern is the hypotension associated with volume loss. The most rapidly available volume expanders are crystalloids to treat hypovolemia.

The admission assessment for a client with acute gastric bleeding indicates blood pressure 82/40, pulse 124, and respiratory rate 26. Which admission request will the nurse implement first?

central and peripheral

The two parts of the nervous system are the:

Intermittent bronchospasm Narrowed airway lemun due to inflammation Stimulation of disease process by allergies Increased eosinophils

Which are characteristics of asthma? (Select all that apply)

one slice whole grain toast with peanut butter, skim milk, orange juice

Which breakfast menu is most appropriate for a patient with diabetes?

A nurse delegates hygiene care for a client who has advanced cirrhosis to an unlicensed nursing personnel (UAP). Which statements should the nurse include when delegating this task to the UAP? (Select all that apply.) a. "Apply lotion to the client's dry skin areas." b. "Use a basin with warm water to bathe the client." c. "For the client's oral care, use a soft toothbrush." d. "Provide clippers so the client can trim the fingernails." e. "Bathe with antibacterial and water-based soaps."

a. "Apply lotion to the client's dry skin areas." c. "For the client's oral care, use a soft toothbrush." d. "Provide clippers so the client can trim the fingernails."

A nurse cares for a client who is prescribed lactulose (Heptalac). The client states, "I do not want to take this medication because it causes diarrhea." How should the nurse respond? a. "Diarrhea is expected; that's how your body gets rid of ammonia." b. "You may take Kaopectate liquid daily for loose stools." c. "Do not take any more of the medication until your stools firm up." d. "We will need to send a stool specimen to the laboratory."

a. "Diarrhea is expected; that's how your body gets rid of ammonia."

A nurse assesses a male client who has symptoms of cirrhosis. Which questions should the nurse ask to identify potential factors contributing to this laboratory result? (Select all that apply.) a. "How frequently do you drink alcohol?" b. "Have you ever had sex with a man?" c. "Do you have a family history of cancer?" d. "Have you ever worked as a plumber?" e. "Were you previously incarcerated?"

a. "How frequently do you drink alcohol?" b. "Have you ever had sex with a man?" e. "Were you previously incarcerated?"

A nurse assesses clients at a community health fair. Which client is at greatest risk for the development of hepatitis B? a. A 20-year-old college student who has had several sexual partners b. A 46-year-old woman who takes acetaminophen daily for headaches c. A 63-year-old businessman who travels frequently across the country d. An 82-year-old woman who recently ate raw shellfish for dinner

a. A 20-year-old college student who has had several sexual partners

An infection control nurse develops a plan to decrease the number of health care professionals who contract viral hepatitis at work. Which ideas should the nurse include in this plan? (Select all that apply.) a. Policies related to consistent use of Standard Precautions b. Hepatitis vaccination mandate for workers in high-risk areas c. Implementation of a needleless system for intravenous therapy d. Number of sharps used in client care reduced where possible e. Postexposure prophylaxis provided in a timely manner

a. Policies related to consistent use of Standard Precautions c. Implementation of a needleless system for intravenous therapy d. Number of sharps used in client care reduced where possible e. Postexposure prophylaxis provided in a timely manner

A nurse assesses a client who is recovering from a paracentesis 1 hour ago. Which assessment finding requires action by the nurse? a. Urine output via indwelling urinary catheter is 20 mL/hr b. Blood pressure increases from 110/58 to 120/62 mm Hg c. Respiratory rate decreases from 18 to 14 breaths/min d. A decrease in the client's weight by 6 kg

a. Urine output via indwelling urinary catheter is 20 mL/hr

After teaching a client who has been diagnosed with hepatitis A, the nurse assesses the client's understanding. Which statement by the client indicates a correct understanding of the teaching? a. "Some medications have been known to cause hepatitis A." b. "I may have been exposed when we ate shrimp last weekend." c. "I was infected with hepatitis A through a recent blood transfusion." d. "My infection with Epstein-Barr virus can co-infect me with hepatitis A."

b. "I may have been exposed when we ate shrimp last weekend."

After teaching a client who has alcohol-induced cirrhosis, a nurse assesses the client's understanding. Which statement made by the client indicates a need for additional teaching? a. "I cannot drink any alcohol at all anymore." b. "I need to avoid protein in my diet." c. "I should not take over-the-counter medications." d. "I should eat small, frequent, balanced meals."

b. "I need to avoid protein in my diet."

A nurse cares for a client with hepatic portal-systemic encephalopathy (PSE). The client is thin and cachectic in appearance, and the family expresses distress that the client is receiving little dietary protein. How should the nurse respond? a. "A low-protein diet will help the liver rest and will restore liver function." b. "Less protein in the diet will help prevent confusion associated with liver failure." c. "Increasing dietary protein will help the client gain weight and muscle mass." d. "Low dietary protein is needed to prevent fluid from leaking into the abdomen."

b. "Less protein in the diet will help prevent confusion associated with liver failure."

A nurse cares for a client with hepatitis C. The client's brother states, "I do not want to contract this infection, so I will not go into his hospital room." How should the nurse respond? a. "If you wear a gown and gloves, you will not get this virus." b. "Viral hepatitis is not spread through casual contact." c. "This virus is only transmitted through a fecal specimen." d. "I can give you an update on your brother's status from here."

b. "Viral hepatitis is not spread through casual contact."

An emergency room nurse assesses a client after a motor vehicle crash. The nurse notices a "steering wheel mark" across the client's chest. Which action should the nurse take? a. Ask the client where in the car he or she was sitting during the crash. b. Assess the client by gently palpating the abdomen for tenderness. c. Notify the laboratory to draw blood for blood type and crossmatch. d. Place the client on the stretcher in reverse Trendelenburg position.

b. Assess the client by gently palpating the abdomen for tenderness.

A nurse cares for a client who is scheduled for a paracentesis. Which intervention should the nurse delegate to an unlicensed assistive personnel (UAP)? a. Have the client sign the informed consent form. b. Assist the client to void before the procedure. c. Help the client lie flat in bed on the right side. d. Get the client into a chair after the procedure.

b. Assist the client to void before the procedure.

A nurse cares for a client who is hemorrhaging from bleeding esophageal varices and has an esophagogastric tube. Which action should the nurse take first? a. Sedate the client to prevent tube dislodgement. b. Maintain balloon pressure at 15 and 20 mm Hg. c. Irrigate the gastric lumen with normal saline. d. Assess the client for airway patency.

d. Assess the client for airway patency.

A nurse assesses a client who is prescribed an infusion of vasopressin (Pitressin) for bleeding esophageal varices. Which clinical manifestation should alert the nurse to a serious adverse effect? a. Nausea and vomiting b. Frontal headache c. Vertigo and syncope d. Mid-sternal chest pain

d. Mid-sternal chest pain

ANS: C The risk of infection is high in the first few months after liver transplant and fever is frequently the only sign of infection.

18. Which finding is most important for the nurse to communicate to the health care provider about a patient who received a liver transplant 1 week ago? a. Dry palpebral and oral mucosa b. Crackles at bilateral lung bases c. Temperature 100.8° F (38.2° C) d. No bowel movement for 4 days

The vertebral column shortens.

An 85-year-old patient comments during his annual physical examination that he seems to be getting shorter as he ages. The nurse should explain that decreased height occurs with aging because:

Asthma Children with asthma usually have these chronic symptoms. Pneumonia appears with an acute onset and fever and general malaise. Bronchiolitis is an acute condition caused by respiratory syncytial virus. Foreign body in the trachea will manifest with acute respiratory distress or failure and maybe stridor.

A child has a chronic, nonproductive cough and diffuse wheezing during the expiratory phase of respiration. This suggests which respiratory condition?

0.9% sodium chloride

A client has a prescription to receive a unit of packed RBCs. The nurse should obtain which of the following IV solutions from the cabinet to hang with the blood product?

A 66-year-old who has a history of cirrhosis The risk of contracting a primary carcinoma of the liver is higher in clients with cirrhosis from any cause. Blunt liver trauma, diabetes mellitus, and chronic malnutrition do not increase a person's risk for developing liver cancer.

A nurse assesses clients on the medical-surgical unit. Which client is at greatest risk for the development of carcinoma of the liver?

ANS: D Hepatitis A is transmitted through the oral-fecal route, and antibody to HAV IgM appears during the acute phase of hepatitis A.

1. A 24-year-old female contracts hepatitis from contaminated food. During the acute (icteric) phase of the patient's illness, the nurse would expect serologic testing to reveal a. antibody to hepatitis D (anti-HDV). b. hepatitis B surface antigen (HBsAg). c. anti-hepatitis A virus immunoglobulin G (anti-HAV IgG). d. anti-hepatitis A virus immunoglobulin M (anti-HAV IgM).

it carries little risk of reaction or exposure to blood-borne infections

The autologous blood transfusion is highly desirable because:

ANS: D TIPS is used to lower pressure in the portal venous system and decrease the risk of bleeding from esophageal varices. Indirect bilirubin level and serum albumin levels are not affected by shunting procedures. TIPS will increase the risk for hepatic encephalopathy.

16. Which finding indicates to the nurse that a patient's transjugular intrahepatic portosystemic shunt (TIPS) placed 3 months ago has been effective? a. Increased serum albumin level b. Decreased indirect bilirubin level c. Improved alertness and orientation d. Fewer episodes of bleeding varices

ANS: C The patient should empty the bladder to decrease the risk of bladder perforation during the procedure. The patient would be positioned in Fowler's position and would not be able to lie flat without compromising breathing.

17. To prepare a 56-year-old male patient with ascites for paracentesis, the nurse a. places the patient on NPO status. b. assists the patient to lie flat in bed. c. asks the patient to empty the bladder. d. positions the patient on the right side.

ANS: B Patients with chronic hepatitis are at higher risk for development of liver cancer, and should be screened for liver cancer every 6 to 12 months. Patients with chronic hepatitis are advised to completely avoid alcohol. There is no hepatitis C vaccine.

40. Which action will the nurse include in the plan of care for a patient who has been diagnosed with chronic hepatitis B? a. Advise limiting alcohol intake to 1 drink daily. b. Schedule for liver cancer screening every 6 months. d. Monitor anti-hepatitis B surface antigen (anti-HBs)

ANS: C all patients who were born between 1945 and 1965 should be screened for hepatitis C because many individuals who are positive have not been diagnosed.

41. A patient born in 1955 had hepatitis A infection 1 year ago. According to CDC , which action should the nurse include in care when the patient is seen for a routine annual physical exam? c. Ask whether the patient has been screened for hepatitis C.

ANS: A The patient's findings of asterixis and lethargy are consistent with grade 2 hepatic encephalopathy. Patients with acute liver failure can deteriorate rapidly from grade 1 or 2 to grade 3 or 4 hepatic encephalopathy and need early transfer to a transplant center.

42. A 23-year-old has been admitted with acute liver failure. Which assessment data are most important for the nurse to communicate to the health care provider? a. Asterixis and lethargy b. Jaundiced sclera and skin c. Elevated total bilirubin level d. Liver 3 cm below costal margin

ANS: B Because ribavirin is teratogenic, the medication will need to be discontinued immediately. Anemia, weight loss, and nausea are common adverse effects of the prescribed regimen and may require actions such as patient teaching, but they would not require immediate cessation of the therapy.

43. A 36-year-old female patient is receiving treatment for chronic hepatitis C with pegylated interferon (PEG-Intron, Pegasys), ribavirin (Rebetol), and telaprevir (Incivek). Which finding is most important to communicate? b. Positive urine pregnancy test c. Hemoglobin level of 10.4 g/dL

You need to get up slowly when youve been lying down or sitting.

A 70-year-old woman tells the nurse that every time she gets up in the morning or after shes been sitting, she gets really dizzy and feels like she is going to fall over. The nurses best response would be:

A The priority intervention for a client with an anorectal abscess focuses on maintaining meticulous perineal hygiene to prevent infection. Comfort measures are also important, but are not as high a priority. Nutrition management and antibiotic teaching may or may not be needed.

A client has an anorectal abscess. Which teaching topic does the nurse address as the priority? a. Perineal hygiene b. Comfort measures c. Nutrition therapy d. Antibiotic use

Make appointments to come get your shot

A client has been prescribed denosumab (Prolia). What instruction about this drug is most appropriate?

B In the first stage of the RPC-IPAA procedure, the temporary ileostomy is created. Because the effluent is caustic, severe skin irritation can occur. The client needs good instruction on ostomy care and comfort measures. Perineal care is not needed because stool drains through the ostomy. Nutrition therapy and relaxation techniques are not as high a priority as preventing skin damage.

A client underwent the first stage of a restorative proctocolectomy with ileo-anal anastomosis (RPC-IPAA). What topic is a high priority for the nurse to teach? a. Perineal care b. Ostomy care c. Nutrition therapy d. Relaxation techniques

D The erythrocyte sedimentation rate (ESR) is an indicator of inflammation, which is elevated during an exacerbation of ulcerative colitis. The normal range for the ESR is 0 to 33 mm/hr. Diarrhea caused by ulcerative colitis will result in loss of potassium and hypokalemia with levels lower than 3.5 mEq/L. Bloody diarrhea will lead to anemia, with hemoglobin levels lower than 12 g/dL in females. The sodium level is normal.

A female client is admitted with an exacerbation of ulcerative colitis. Which laboratory value does the nurse correlate with this condition? a. Potassium, 5.5 mEq/L b. Hemoglobin, 14.2 g/dL c. Sodium, 144 mEq/L d. Erythrocyte sedimentation rate (ESR), 55 mm/hr

has brisk capillary refill

A goal of ensuring adequate tissue perfusion has been established for a client with anemia. What would be a desired outcome for this goal?

Raise the arm above the level of the heart.

A home health nurse assesses a client with diabetes who has a new cast on the arm. The nurse notes the clients fingers are pale, cool, and slightly swollen. Which action should the nurse take first?

It is not necessary for this client to have a snack because Lantus insulin is absorbed very slowly over 24 hours and doesn't have a peak.

A nurse administers 15 units of Lantus insulin at 2100 hour to a Hispanic client when the client's fingerstick BG reading was 110mg/dL. At 2300 hours, a nursing assistant reports to the nurse that an evening snack was not given because the client was sleeping. Which instruction by the nurse is most appropriate?

ANS: C - The nurse expects high-pitched, rushing bowel sounds due to narrowing of the bowel lumen in Crohns disease. A positive Murphys sign is indicative of gallbladder disease, and rebound tenderness often indicates peritonitis. Dullness in the lower abdominal quadrants and hypoactive bowel sounds are not commonly found with Crohns disease. Nightly worsening of abdominal cramping is not consistent with Crohns disease.

A nurse assesses a client who is hospitalized with an exacerbation of Crohns disease. Which clinical manifestation should the nurse expect to find? a. Positive Murphys sign with rebound tenderness to palpitation b. Dull, hypoactive bowel sounds in the lower abdominal quadrants c. High-pitched, rushing bowel sounds in the right lower quadrant d. Reports of abdominal cramping that is worse at night

ANS: A - The nurse should assess the stoma for color and contact the health care provider if the stoma is pale, bluish, or dark. The nurse should expect the client to have an intact ostomy pouch with dark green liquid stool that may contain some blood.

A nurse assesses a client who is recovering from an ileostomy placement. Which clinical manifestation should alert the nurse to urgently contact the health care provider? a. Pale and bluish stoma b. Liquid stool c. Ostomy pouch intact d. Blood-smeared output

ANS: A - The presence of strictures predisposes the client to intestinal obstruction. Abdominal distention may indicate that the client has developed an obstruction of the large bowel, and the clients provider should be notified right away. Low-grade fever, bloody diarrhea, and abdominal cramps are common symptoms of Crohns disease.

A nurse assesses a client with Crohns disease and colonic strictures. Which clinical manifestation should alert the nurse to urgently contact the health care provider? a. Distended abdomen b. Temperature of 100.0 F (37.8 C) c. Loose and bloody stool d. Lower abdominal cramps

Assess the clients bowel sounds.

A nurse assesses a client with a mechanical bowel obstruction who reports intermittent abdominal pain. An hour later the client reports constant abdominal pain. Which action should the nurse take next?

Body mass index of 46 Pregnant with twins Glycosylated hemoglobin level of 15% Obesity, pregnancy, and diabetes are all risk factors for the development of cholelithiasis. A diet low in saturated fats and moderate alcohol intake may decrease the risk. Although metabolic syndrome is a precursor to diabetes, it is not a risk factor for cholelithiasis. The client should be informed of the connection.

A nurse assesses a client with cholelithiasis. Which assessment findings should the nurse identify as contributors to this client's condition? (Select all that apply.)

ANS: A, B, E - A client with peritonitis may present with a distended abdomen, diminished bowel sounds, inability to pass flatus or feces, tachycardia, and decreased urine output secondary to dehydration. Bradycardia and hyperactive bowel sounds are not associated with peritonitis.

A nurse assesses a client with peritonitis. Which clinical manifestations should the nurse expect to find? (Select all that apply.) a. Distended abdomen b. Inability to pass flatus c. Bradycardia d. Hyperactive bowel sounds e. Decreased urine output

ANS: A, B, D - Lower GI bleeding can lead to erosion of the bowel wall. Abscesses are localized pockets of infection that develop in the ulcerated bowel lining. Nonmechanical bowel obstruction is paralysis of the colon that results from colorectal cancer. When the inflammation is transmural, fistulas can occur between the bowel and bladder resulting in pyuria and fecaluria. Paralysis of the colon causing dilation and subsequent colonic ileus is known as a toxic megacolon.

A nurse assesses a client with ulcerative colitis. Which complications are paired correctly with their physiologic processes? (Select all that apply.) a. Lower gastrointestinal bleeding Erosion of the bowel wall b. Abscess formation Localized pockets of infection develop in the ulcerated bowel lining c. Toxic megacolon Transmural inflammation resulting in pyuria and fecaluria d. Nonmechanical bowel obstruction Paralysis of colon resulting from colorectal cancer e. Fistula Dilation and colonic ileus caused by paralysis of the colon

Pulse decreased from 100 to 80 beats/min Beta-blockers block the stimulation of beta1-adrenergic receptors. They block the sympathetic (fight-or-flight) response and decrease the heart rate (HR). The beta-blocker will decrease HR and blood pressure, increasing ventricular filling time. It usually does not have effects on beta2-adrenergic receptor sites. Cardiac output will drop because of decreased HR.

A nurse assesses a patient after administering a prescribed beta-blocker. Which assessment would the nurse expect to find?

Allow the client to be as independent as possible with activities

A nurse delegates care for a client with Parkinson disease to an unlicensed assistive personnel (UAP). Which statement should the nurse include when delegating this clients care?

A 27-year-old client with a heart rate of 120 beats/min Tachycardia can indicate hypoxemia as the body tries to circulate the oxygen that is available. A barrel chest is not an emergency finding. Likewise, a pulse oximetry level of 92% is not considered an acute finding. The expiratory phase is expected to be longer than the inspiratory phase in someone with airflow limitation.

A nurse assesses several clients who have a history of asthma. Which client should the nurse assess first?

Skeletal traction will assist in realigning your fractured bone.

A nurse cares for a client placed in skeletal traction. The client asks, What is the primary purpose of this type of traction? How should the nurse respond?

You need a new cast now that the swelling is decreased.

A nurse cares for a client who had a wrist cast applied 3 days ago. The client states, The cast is loose enough to slide off. How should the nurse respond?

ANS: A - Metronidazole is the drug of choice for a Giardia infection. Ciprofloxacin and ceftriaxone are antibiotics used for bacterial infections. Sulfasalazine is used for ulcerative colitis and Crohns disease.

A nurse cares for a client who has a Giardia infection. Which medication should the nurse anticipate being prescribed for this client? a. Metronidazole (Flagyl) b. Ciprofloxacin (Cipro) c. Sulfasalazine (Azulfidine) d. Ceftriaxone (Rocephin)

-Assess for proper placement of the tube every 4 hours -Disconnect suction when auscultating bowel peristalsis. -Monitor the clients skin around the tube site for irritation.

A nurse cares for a client who has a nasogastric (NG) tube. Which actions should the nurse take? (Select all that apply.)

Provide a low-sodium diet. A low-sodium diet is one means of controlling abdominal fluid collection. Monitoring intake and output does not control fluid accumulation, nor does weighing the client. These interventions merely assess or monitor the situation. Increasing fluid intake would not be helpful.

A nurse cares for a client who has cirrhosis of the liver. Which action should the nurse take to decrease the presence of ascites?

ANS: D - Severe infection with C. botulinum can lead to respiratory failure, so assessments of oxygen saturation and respiratory rate are of high priority for clients with suspected C. botulinum infection. The other assessments may be completed after the respiratory system has been assessed.

A nurse cares for a client who has food poisoning resulting from a Clostridium botulinum infection. Which assessment should the nurse complete first? a. Heart rate and rhythm b. Bowel sounds c. Urinary output d. Respiratory rate

Check the patient's blood pressure. Because the patient's history suggests that fluid volume deficit may be a problem, assessment for adequate circulation is the highest priority. The other actions are also appropriate, but are not as essential as determining the patient's perfusion status.

A patient comes to the clinic complaining of frequent, watery stools for the last 2 days. Which action should the nurse take first?

ANS: C - Asacol is the oral formula for mesalamine and is produced as an enteric-coated pill that should not be crushed, chewed, or broken. Asacol is not available as a suspension or elixir. If the client is unable to swallow the Asacol pill, a mesalamine enema (Rowasa) may be administered instead, with a providers order.

A nurse cares for a client who is prescribed mesalamine (Asacol) for ulcerative colitis. The client states, I am having trouble swallowing this pill. Which action should the nurse take? a. Contact the clinical pharmacist and request the medication in suspension form. b. Empty the contents of the capsule into applesauce or pudding for administration. c. Ask the health care provider to prescribe the medication as an enema instead. d. Crush the pill carefully and administer it in applesauce or pudding.

ANS: B - Social anxiety and apprehension are common in clients with a new ileostomy. The nurse should encourage the client to discuss concerns. The nurse should not minimize the clients concerns or provide false reassurance.

A nurse cares for a client with a new ileostomy. The client states, I dont think my friends will accept me with this ostomy. How should the nurse respond? a. Your friends will be happy that you are alive. b. Tell me more about your concerns. c. A therapist can help you resolve your concerns. d. With time you will accept your new body.

"Have you passed any flatus or moved your bowels?" Paralytic ileus is a common complication of acute pancreatitis. The client should not eat until this has resolved. Bowel sounds and decreased pain are not reliable indicators of peristalsis. Instead, the nurse should assess for passage of flatus or bowel movement.

A nurse cares for a client with acute pancreatitis. The client states, "I am hungry." How should the nurse reply?

"Share any thoughts and feelings that cause you to limit social activities." Many clients with moderate to severe COPD become socially isolated because they are embarrassed by frequent coughing and mucus production. They also can experience fatigue, which limits their activities. The nurse needs to encourage the client to verbalize thoughts and feelings so that appropriate interventions can be selected. Joining a support group would not decrease feelings of social isolation if the client does not verbalize feelings. Antianxiety agents will not help the client with social isolation. Encouraging a client to participate in activities without verbalizing concerns also would not be an effective strategy for decreasing social isolation.

A nurse cares for a client with chronic obstructive pulmonary disease (COPD). The client states that he no longer enjoys going out with his friends. How should the nurse respond?

Elevation of the left heel off the bed

A patient had a CVA (stroke) 2 days ago, resulting in decreased mobility to her left side. During the assessment, the nurse discovers a stage I pressure area on the patient's left heel. What is the initial treatment for this pressure ulcer?

chest x-ray to show the tip of the catheter

A patient has a PICC placed at the bedside. Before the first use, how is the placement verified?

ANS: A - Clients with ulcerative colitis often express that the disorder is disruptive to their lives. Stress factors can increase symptoms. These factors should be identified so that the client will have more control over his or her condition. Prescription medications and anorexia will not eliminate exacerbations. Although a therapist may assist the client, this is not an appropriate response.

A nurse cares for a client with ulcerative colitis. The client states, I feel like I am tied to the toilet. This disease is controlling my life. How should the nurse respond? a. Lets discuss potential factors that increase your symptoms. b. If you take the prescribed medications, you will no longer have diarrhea. c. To decrease distress, do not eat anything before you go out. d. You must retake control of your life. I will consult a therapist to help.

Assess the patient for airway patency. Maintaining airway patency is the primary nursing intervention for this patient. The nurse suctions oral secretions to prevent aspiration and occlusion of the airway. The patient usually is intubated and mechanically ventilated during this treatment. The patient would be sedated, balloon pressure would be maintained between 15 and 20 mm Hg, and the lumen can be irrigated with saline or tap water. However, these are not a higher priority than airway patency.

A nurse cares for a patient who is hemorrhaging from bleeding esophageal varices and has an esophagogastric tube. What action would the nurse take first?

ANS: C - The ostomy nurse is a valuable resource for clients, providing suggestions for supplies and methods to manage the ostomy. A larger dress size will not necessarily help hide the ostomy appliance. Avoiding broccoli and carbonated drinks does not offer reassurance for the client. Ileostomies have an almost constant liquid effluent, so pouch removal during the prom is not feasible.

A nurse cares for a teenage girl with a new ileostomy. The client states, I cannot go to prom with an ostomy. How should the nurse respond? a. Sure you can. Purchase a prom dress one size larger to hide the ostomy appliance. b. The pouch wont be as noticeable if you avoid broccoli and carbonated drinks prior to the prom. c. Lets talk to the enterostomal therapist about options for ostomy supplies and dress styles. d. You can remove the pouch from your ostomy appliance when you are at the prom so that it is less noticeable.

ANS: B - Dehydration caused by diarrhea can occur quickly in older clients with Salmonella food poisoning, so maintenance of fluid balance is a high priority. Monitoring vital signs and providing perineal care are important nursing actions but are of lower priority than fluid replacement. The nurse should teach the client about proper hand hygiene to prevent the spread of infection, and preparation of food and beverages to prevent contamination.

A nurse cares for an older adult client who has Salmonella food poisoning. The clients vital signs are heart rate: 102 beats/min, blood pressure: 98/55 mm Hg, respiratory rate: 22 breaths/min, and oxygen saturation: 92%. Which action should the nurse complete first? a. Apply oxygen via nasal cannula. b. Administer intravenous fluids. c. Provide perineal care with a premedicated wipe. d. Teach proper food preparation to prevent contamination.

Planning to teach about bisphosphonates

A patient has a bone density score of -2.8. What action by the nurse is best?

Assess the airway. All interventions are important for patients with respiratory acidosis; this is indicated by the ABGs. However, the priority is assessing and maintaining an airway. Without a patent airway, other interventions will not be helpful.

A nurse evaluates a patient's arterial blood gas values (ABGs): pH 7.30, PaO2 86 mm Hg, PaCO2 55 mm Hg, and HCO3- 22 mEq/L (22 mmol/L). Which intervention does the nurse implement first?

"I will take this medication on an empty stomach." INH needs to be taken on an empty stomach, either 1 hour before or 2 hours after meals. Extra vitamin B needs to be taken while on the drug. Staining of contact lenses commonly occurs while taking rifampin (Rifadin).

A nurse has educated a client on isoniazid (INH). What statement by the client indicates teaching has been effective?

"Do you have any chronic breathing problems?" The normal chest has a lateral diameter that is twice as large as the AP diameter. When the AP diameter approaches or exceeds the lateral diameter, the patient is said to have a barrel chest. Most commonly, barrel chest occurs as a result of a long-term chronic airflow limitation problem, such as chronic obstructive pulmonary disease or severe chronic asthma. It can also be seen in people who have lived at a high altitude for many years. Therefore, an AP chest diameter that is the same as the lateral chest diameter would be rechecked but is not as indicative of underlying disease processes as an AP diameter that exceeds the lateral diameter. Medications, herbal supplements, and aerobic exercise are not associated with a barrel chest. Although occupation and hobbies may expose a patient to irritants that can cause chronic lung disorders and barrel chest, asking about chronic breathing problems is more direct and would be asked first.

A nurse observes that a patient's anteroposterior (AP) chest diameter is the same as the lateral chest diameter. Which question would the nurse ask the patient in response to this finding?

ANS: B - Protecting the clients skin is the priority action for a client who has a heavily draining fistula. Intestinal fluid enzymes are caustic and can cause skin breakdown or fungal infections if the skin is not protected. The plan of care for a client who has Crohns disease includes adequate nutrition focused on high-calorie, high-protein, high-vitamin, and low-fiber meals, antibiotic administration, and glucocorticoids.

A nurse plans care for a client with Crohns disease who has a heavily draining fistula. Which intervention should the nurse indicate as the priority action in this clients plan of care? a. Low-fiber diet b. Skin protection c. Antibiotic administration d. Intravenous glucocorticoids

Meperidine (Demerol) 50 mg IV every 4 hours

A nurse reviews prescriptions for an 82-year-old client with a fractured left hip. Which prescription should alert the nurse to contact the provider and express concerns for client safety?

encourage the patient to drink more liquids

A patient has thick tenacious secretions. Which should the nurse do to liquefy the patient's respiratory secretions?

ANS: A, C, D, E - Salmonella is usually contracted via contaminated eggs, beef, poultry, and green leafy vegetables. It is not transmitted through water in garden hoses or pools. Clients should wash leafy vegetables well, wash hands before and after using the restroom, make sure meat and eggs are cooked properly, and, because it can be transmitted by flies, keep flies off of food.

A nurse teaches a client how to avoid becoming ill with Salmonella infection again. Which statements should the nurse include in this clients teaching? (Select all that apply.) a. Wash leafy vegetables carefully before eating or cooking them. b. Do not ingest water from the garden hose or the pool. c. Wash your hands before and after using the bathroom. d. Be sure meat is cooked to the proper temperature. e. Avoid eating eggs that are sunny side up or undercooked.

ANS: A, D - Washing hands after contact with animals and using separate cutting boards for meat and other foods will help prevent E. coli infection. The other statements are not related to preventing E. coli infection.

A nurse teaches a community group ways to prevent Escherichia coli infection. Which statements should the nurse include in this groups teaching? (Select all that apply.) a. Wash your hands after any contact with animals. b. It is not necessary to buy a meat thermometer. c. Stay away from people who are ill with diarrhea. d. Use separate cutting boards for meat and vegetables. e. Avoid swimming in backyard pools and using hot tubs.

Daily weight Daily weight is the most easily obtained and accurate means of assessing volume status. Skin turgor varies considerably with age. Considerable excess fluid volume may be present before fluid moves into the interstitial space and causes edema. Although very important, hourly urine outputs do not take account of fluid intake or of fluid loss through insensible loss, sweating, or loss from the gastrointestinal tract or wounds.

A patient is admitted for hypovolemia associated with multiple draining wounds. Which assessment would be the most accurate way for the nurse to evaluate fluid balance?

subcutaneous low-molecular-weight heparins

A patient is admitted to the hospital with deep vein thrombosis (DVT). Which drugs are preferred for treatment and prevention of DVT?

Adduction

A patient is being assessed for range-of-joint movement. The nurse asks him to move his arm in toward the center of his body. This movement is called:

Allow the patient's family to remain at the bedside. Pull the curtains around the bed to provide privacy. Provide warm blankets or cool washcloths as desired. Allow the patient to assume a position of comfort.

A patient is clearly uncomfortable and anxious in the preoperative holding room waiting for emergent abdominal surgery. What actions can the nurse perform to increase comfort? (Select all that apply.)

Place the patient in a high Fowler's position and start O2

A patient presents to the walk-in clinic with extremely labored breathing and a history of asthma that is unresponsive to prescribed inhalers or medications. What is the first priority nursing action?

Flexion

A patient tells the nurse that she is having a hard time bringing her hand to her mouth when she eats or tries to brush her teeth. The nurse knows that for her to move her hand to her mouth, she must perform which movement?

Respiratory acidosis, partially compensated

A patient with ABG values of pH7.34, PaCO2 44, HCO3 28, the nurse would analyze these findings to indicate?

After teaching a client who is recovering from laparoscopic cholecystectomy surgery, the nurse assesses the client's understanding. Which statement made by the client indicates a correct understanding of the teaching? a. "Drinking at least 2 liters of water each day is suggested." b. "I will decrease the amount of fatty foods in my diet." c. "Drinking fluids with my meals will increase bloating." d. "I will avoid concentrated sweets and simple carbohydrates."

ANS: B After cholecystectomy, clients need a nutritious diet without a lot of excess fat; otherwise a special diet is not recommended for most clients. Good fluid intake is healthy for all people but is not related to the surgery. Drinking fluids between meals helps with dumping syndrome, which is not seen with this procedure. Restriction of sweets is not required.

A nurse plans care for a client with acute pancreatitis. Which intervention should the nurse include in this client's plan of care to reduce discomfort? a. Administer morphine sulfate intravenously every 4 hours as needed. b. Maintain nothing by mouth (NPO) and administer intravenous fluids. c. Provide small, frequent feedings with no concentrated sweets. d. Place the client in semi-Fowler's position with the head of bed elevated.

ANS: B The client should be kept NPO to reduce GI activity and reduce pancreatic enzyme production. IV fluids should be used to prevent dehydration. The client may need a nasogastric tube. Pain medications should be given around the clock and more frequently than every 4 to 6 hours. A fetal position with legs drawn up to the chest will promote comfort.

"I need to avoid protein in my diet." Based on the degree of liver involvement and decreased function, protein intake may have to be decreased. However, some protein is necessary for the synthesis of albumin and normal healing. The other statements indicate accurate understanding of self-care measures for this client.

After teaching a client who has alcohol-induced cirrhosis, a nurse assesses the client's understanding. Which statement made by the client indicates a need for additional teaching?

ANS: C - Laxatives are not recommended for clients with diverticulitis because they can increase pressure in the bowel, causing additional outpouching of the lumen. Exercise and a high-fiber diet are recommended for clients with diverticulitis because they promote regular bowel function. Using the leg muscles rather than the back for lifting prevents abdominal straining.

After teaching a client who has diverticulitis, a nurse assesses the clients understanding. Which statement made by the client indicates a need for additional teaching? a. Ill ride my bike or take a long walk at least three times a week. b. I must try to include at least 25 grams of fiber in my diet every day. c. I will take a laxative nightly at bedtime to avoid becoming constipated. d. I should use my legs rather than my back muscles when I lift heavy objects.

ANS: B - Adalimumab (Humira) is an immune modulator that must be given via subcutaneous injection. It does not need to be given with food or milk. Nausea and vomiting are two common side effects. Adalimumab can cause immune suppression, so clients receiving the medication should avoid large crowds and people who are sick, and should practice good handwashing.

After teaching a client who is prescribed adalimumab (Humira) for severe ulcerative colitis, the nurse assesses the clients understanding. Which statement made by the client indicates a need for additional teaching? a. I will avoid large crowds and people who are sick. b. I will take this medication with my breakfast each morning. c. Nausea and vomiting are common side effects of this drug. d. I must wash my hands after I play with my dog.

ANS: B The liver is often injured by a steering wheel in a motor vehicle crash. Because the client's chest was marked by the steering wheel, the nurse should perform an abdominal assessment. Assessing the client's position in the crash is not needed because of the steering wheel imprint. The client may or may not need a blood transfusion. The client does not need to be in reverse Trendelenburg position.

An emergency room nurse assesses a client after a motor vehicle crash. The nurse notices a "steering wheel mark" across the client's chest. Which action should the nurse take? a. Ask the client where in the car he or she was sitting during the crash. b. Assess the client by gently palpating the abdomen for tenderness. c. Notify the laboratory to draw blood for blood type and crossmatch. d. Place the client on the stretcher in reverse Trendelenburg position.

the presence of palpable peripheral pulses

An indicator of normal arterial circulation is:

Change in behavior Older people have an age-related decrease in immune system functioning and may not show classic signs of infection such as increased white blood cell count, fever and chills, or obvious localized signs of infection. A change in behavior often signals an infection or onset of other illnesses in the older patient. Tolerating increasing activity is a positive sign.

An older patient is hospitalized after an operation. When assessing the patient for postoperative infection, the nurse places priority on which assessment?

Introducing one's self Correct positioning Providing warmth Remaining present

What actions by the circulating nurse are important to promote patient comfort? (Select all that apply.)

Valvular disease Excessive alcohol use Hypertension Diabetes Mellitus

What are the risk factors for atrial fibrillation? (Select all that apply)

atrial depolarization

What does the P wave in an ECG represent?

ventricular repolarization

What does the T wave on the ECG represent?

Presence of the Chvostek's sign The patient's symptoms indicate possible hypocalcemia, which can occur secondary to parathyroid injury/removal during thyroidectomy. There is no indication of a need to check the potassium level, the thyroid hormone level, or for bleeding

Following a thyroidectomy, a patient complains of "a tingling feeling around my mouth." Which assessment should the nurse complete immediately?

Wound will close with no evidence of infection within 6 weeks.

For the client with a stage IV pressure ulcer, what would an applicable patient goal/outcome be?

Intermittent claudication

For which symptom so most patients seek medical attention when they have peripheral arterial disease?

7.35-7.45

Normal value for arterial blood pH range is from:

explain the rules of fire safety to the patient

Oxygen therapy via nasal cannula is ordered for a patient. Which should the nurse do first?

"We know that there can be an association between Crohn's disease and chronic gastritis, but Crohn's does not directly cause acute gastritis to develop." Correct: This is the only accurate statement. Crohn's disease may be an underlying disease process when chronic gastritis develops, but not when acute gastritis occurs.

The client has a long-term history of Crohn's disease and has recently developed acute gastritis. The client asks the nurse whether Crohn's disease was a direct cause of the gastritis. What is the nurse's best response?

D Whole grains (rice pilaf), uncooked fruits and vegetables (salad, fresh fruit cup), and high-fiber foods (cup of bean soup) should be avoided with a low-residue diet. Canned or cooked vegetables are appropriate. Apple juice does not contain fiber and is acceptable for a low-residue diet.

The nurse helps a client with diverticular disease choose appropriate dinner options. Which menu selections are most appropriate? a. Roasted chicken, rice pilaf, cup of coffee with cream b. Spaghetti with meat sauce, fresh fruit cup, hot tea with lemon c. Chicken Caesar salad, cup of bean soup, glass of low-fat milk d. Baked fish with steamed asparagus, dinner roll with butter, glass of apple juice

C The nurse recognizes that the client may be expressing feelings of grief. The nurse should encourage the client to verbalize feelings and identify fears to move the client through the phases of the grief process. A psychiatric consult is not appropriate for the client. The nurse should not brush aside the client's feelings with a generalization about cancer prognosis and treatment. The nurse should not ignore the client's withdrawal behavior.

The nurse is caring for a client who has been newly diagnosed with colon cancer. The client has become withdrawn from family members. Which strategy does the nurse use to assist the client at this time? a. Ask the health care provider for a psychiatric consult for the client. b. Explain the improved prognosis for colon cancer with new treatment. c. Encourage the client to verbalize feelings about the diagnosis. d. Allow the client to remain withdrawn as long as he or she wishes.

B On noticing the ecchymotic areas, the nurse should check to see if abdominal guarding or rigidity is present; this could indicate major organ injury. The nurse should then notify the provider. Measuring abdominal girth or asking about seating in the car is not appropriate at this time. Laboratory test results can be checked after assessment for abdominal guarding or rigidity.

The nurse is caring for a client who is brought to the emergency department following a motor vehicle crash. The nurse notes that the client has ecchymotic areas across the lower abdomen. Which is the priority action of the nurse? a. Measure the client's abdominal girth. b. Assess for abdominal guarding or rigidity. c. Check the client's hemoglobin and hematocrit. d. Ask whether the client was riding in the front or back seat of the car.

B Campylobacter gastroenteritis causes foul-smelling diarrhea with up to 20 to 30 stools per day for 7 days. Both RBCs and WBCs are present in a Gram stain of the stools. Infection with Clostridium causes not diarrhea, but constipation, paralysis, and respiratory failure. H. pylori is a common cause of gastric ulcers, not gastroenteritis. Norwalk virus produces milder illness with diarrhea and vomiting.

The nurse is caring for a client who is having approximately 20 foul-smelling stools each day. Laboratory Gram stain testing indicates the presence of white blood cells (WBCs) and red blood cells (RBCs) in the stool. Which organism does the nurse expect to see in the culture report? a. Helicobacter pylori b. Campylobacter jejuni c. Clostridium botulinum d. Norwalk virus

C The nurse expects high-pitched, rushing bowel sounds due to narrowing of the bowel lumen in Crohn's disease. Dullness in the lower abdominal quadrants and hypoactive bowel sounds are not commonly found with Crohn's disease. Nightly worsening of abdominal cramping is not consistent with Crohn's disease. A positive Murphy's sign is indicative of gallbladder disease, and rebound tenderness often indicates peritonitis.

The nurse is caring for a client who is hospitalized with exacerbation of Crohn's disease. What does the nurse expect to find during the physical assessment? a. Positive Murphy's sign with rebound tenderness b. Dullness in the lower abdominal quadrants c. High-pitched, rushing bowel sounds in the right lower quadrant d. Abdominal cramping that the client says is worse at night

C Client learning goals must be measurable and objective with a time frame, so the nurse can determine whether they have been met. When the goal is to have the client demonstrate a particular skill, the nurse can easily determine whether the goal was met. The specific time frame of "by discharge" is easily measurable also. The other goals are all subjective and cannot be measured objectively. The first two options do not have time frames. "Tomorrow" is a vague time frame.

The nurse is caring for a client who just had colon resection surgery with a new colostomy. Which teaching objective does the nurse include in the client's plan of care? a. Understanding colostomy care and lifestyle implications b. Learning how to change the appliance independently c. Demonstrating the correct way to change the appliance by discharge d. Not being afraid to handle the ostomy appliance tomorrow

hypoxemia

__________ is a decrease in blood oxygen concentration.

Notify the health care provider immediately.

The nurse is caring for a client with peptic ulcer disease who reports sudden onset of sharp abdominal pain. On palpation, the clients abdomen is tense and rigid. What action takes priority?

B Adalimumab (Humira) is an immune modulator that must be given via subcutaneous injection. It does not need to be given with food or milk. Nausea and vomiting are two common side effects. Adalimumab can cause immune suppression, so clients receiving the medication should avoid large crowds and people who are sick, and should practice good handwashing.

The nurse is caring for a client with severe ulcerative colitis who has been prescribed adalimumab (Humira). Which client statement indicates that additional teaching about the medication is needed? a. "I will avoid large crowds and people who are sick." b. "I will take this medication with food or milk." c. "Nausea and vomiting are common side effects." d. "I will wash my hands after I play with my dog."

Epiphyses

The nurse is explaining the mechanism of the growth of long bones to a mother of a toddler. Where does lengthening of the bones occur?

C Laxatives are not recommended for clients with diverticulitis because they can increase pressure in the bowel, causing additional outpouching of the lumen. Exercise and a high-fiber diet are recommended for clients with diverticulitis because they promote regular bowel function. Using the leg muscles rather than the back for lifting prevents abdominal straining.

The nurse is preparing a client with diverticulitis for discharge from the hospital. Which statement by the client indicates that additional teaching is needed? a. "I will ride my bike or take a long walk at least three times a week." b. "I will try to include at least 25 g of fiber in my diet every day." c. "I will take a senna laxative at bedtime to avoid becoming constipated." d. "I will use my legs rather than my back muscles when I lift heavy objects."

Visible defect with an external saclike protrusion containing meninges, spinal fluid, and nerves

The nurse is preparing to admit a newborn with myelomeningocele to the neonatal intensive care nursery. Which describes this newborns defect?

D Colon cancer is rare before the age of 40, but its incidence increases rapidly with advancing age. Fast food tends to be high in fat and low in fiber, increasing the risk for colon cancer. Irritable bowel syndrome, a heavy workload, and coffee intake do not increase the risk for colon cancer. A BMI of 19.2 is within normal limits.

The nurse is screening clients at a community health fair. Which client is at highest risk for development of colorectal cancer? a. Young adult who drinks eight cups of coffee every day b. Middle-aged client with a history of irritable bowel syndrome c. Older client with a BMI of 19.2 who works 65 hours per week d. Older client who travels extensively and eats fast food frequently

Oxygenate with 100% oxygen and monitor the patient

The nurse is suctioning the secretions from a patient's endotracheal tube with Asthma. the patient demonstrates a vagal response by a drop in heart rate to 54 beats /minute and a drop in blood pressure to 90/50 mmHg. After stopping suctioning, what is the nurse's priority action?

A patient with a respiratory rate of 38/minute A respiratory rate of 38/minute indicates severe respiratory distress, and the patient needs immediate assessment and intervention to prevent possible respiratory arrest. The other patients also need assessment as soon as possible, but they do not need to be assessed as urgently as the tachypneic patient.

The nurse receives a change-of-shift report on the following patients with chronic obstructive pulmonary disease (COPD). Which patient should the nurse assess first?

Use the short-acting beta-adrenergic medication

The nurse teaches a patient with asthma to perform which intervention before exercising?

A nurse obtains a client's health history at a community health clinic. Which statement alerts the nurse to provide health teaching to this client? a. "I drink two glasses of red wine each week." b. "I take a lot of Tylenol for my arthritis pain." c. "I have a cousin who died of liver cancer." d. "I got a hepatitis vaccine before traveling."

b. "I take a lot of Tylenol for my arthritis pain."

A nurse assesses a client who has liver disease. Which laboratory findings should the nurse recognize as potentially causing complications of this disorder? (Select all that apply.) a. Elevated aspartate transaminase b. Elevated international normalized ratio (INR) c. Decreased serum globulin levels d. Decreased serum alkaline phosphatase e. Elevated serum ammonia f. Elevated prothrombin time (PT)

b. Elevated international normalized ratio (INR) e. Elevated serum ammonia f. Elevated prothrombin time (PT)

A nurse cares for a client who has cirrhosis of the liver. Which action should the nurse take to decrease the presence of ascites? a. Monitor intake and output. b. Provide a low-sodium diet. c. Increase oral fluid intake. d. Weigh the client daily.

b. Provide a low-sodium diet.

After teaching a client who has plans to travel to a non-industrialized country, the nurse assesses the client's understanding regarding the prevention of viral hepatitis. Which statement made by the client indicates a need for additional teaching? a. "I should drink bottled water during my travels." b. "I will not eat off another's plate or share utensils." c. "I should eat plenty of fresh fruits and vegetables." d. "I will wash my hands frequently and thoroughly."

c. "I should eat plenty of fresh fruits and vegetables."

With dehiscence, there is a separation of one or more layers of wound tissue; evisceration involves the protrusion of internal viscera from the incision site.

Which of the following describes the difference between dehiscence and evisceration?

blood

Which of the following is not involved in maintaining acid-base balance?

2. Alcohol is a gastric irritant that should be eliminated from the intake of the client with peptic ulcer disease. Analgesics are not used to control ulcer pain; many analgesics are gastric irritants. The client's hemoglobin and hematocrit typically do not need to be monitored every 3 months, unless gastrointestinal bleeding is suspected. The client can maintain an active lifestyle and does not need to eliminate contact sports as long as they are not stress-inducing.

Which of the following would be an expected outcome for a client with peptic ulcer disease? The client will: 1. Demonstrate appropriate use of analgesics to control pain. 2. Explain the rationale for eliminating alcohol from the diet. 3. Verbalize the importance of monitoring hemoglobin and hematocrit every 3 months. 4. Eliminate contact sports from his or her lifestyle.

contusion

Which of the following wounds classifies as a closed wound?

The peripheral nerves carry input to the central nervous system by afferent fibers and away from the central nervous system by efferent fibers.

Which of these statements about the peripheral nervous system is correct?

Which patient has the greatest risk for pulmonary embolus related to a venous disorder?

Which patient has the greatest risk for pulmonary embolus related to a venous disorder?

ANS: C The pressure-relieving mattress will decrease the risk for skin breakdown for this patient. Adequate dietary protein intake is necessary in patients with ascites to improve oncotic pressure.

15. A 38-year-old patient with cirrhosis has ascites and 4+ edema of the feet and legs. Which nursing action will be included in the plan of care? a. Restrict daily dietary protein intake. b. Reposition the patient every 4 hours. c. Place the patient on a pressure-relieving mattress.

ANS: B The presence of surface antibody to HBV (anti-HBs) is a marker of a positive response to the vaccine. The other laboratory values indicate current infection with HBV.

2. Administration of hepatitis B vaccine to a healthy 18-year-old patient has been effective when a specimen of the patient's blood reveals a. HBsAg. b. anti-HBs. c. anti-HBc IgG. d. anti-HBc IgM.

ANS: B Maintaining adequate nutritional intake is important for regeneration of hepatocytes. Interferon and antivirals may be used for chronic hepatitis B, but they are not prescribed for acute hepatitis B infection. Rest is recommended.

4. The nurse will plan to teach the patient diagnosed with acute hepatitis B about a. side effects of nucleotide analogs. b. measures for improving the appetite. c. ways to increase activity and exercise. d. administering α-interferon (Intron A).

ANS: A Therapy with ribavirin and α-interferon may cause leukopenia. The other problems are not associated with this drug therapy.

5. The nurse administering α-interferon and ribavirin (Rebetol) to a patient with chronic hepatitis C will plan to monitor for a. leukopenia. b. hypokalemia. c. polycythemia. d. hypoglycemia.

ANS: B Any patient with a history of IV drug use should be tested for hepatitis C.

6. Which information given by a 70-year-old patient during a health history indicates to the nurse that the patient should be screened for hepatitis C? b. The patient used IV drugs about 20 years ago.

ANS: B The patient's symptoms, lack of antibodies for hepatitis, and the abrupt onset of symptoms suggest toxic hepatitis, which can be caused by commonly used over-the-counter drugs such as acetaminophen (Tylenol).

7. A 55-year-old patient admitted with an abrupt onset of jaundice and nausea has abnormal liver function studies but serologic testing is negative for viral causes of hepatitis. Which question by the nurse is most appropriate? b. "Do you use any over-the-counter drugs?"

ANS: D Albumin level The low oncotic pressure caused by hypoalbuminemia is a major pathophysiologic factor in the development of edema. The other parameters should also be monitored, but they are not directly associated with the patient's current symptoms.

8. Which data will the nurse monitor in relation to the 4+ pitting edema assessed in a patient with cirrhosis? a. Hemoglobin b. Temperature c. Activity level d. Albumin level

ANS: B The disease progression can be stopped or reversed by alcohol abstinence. The other interventions may be used when cirrhosis becomes more severe to decrease symptoms or complications, but the priority for this patient is to stop the progression of the disease.

9. Which topic is most important to include in patient teaching for a 41-year-old patient diagnosed with early alcoholic cirrhosis? a. Maintaining good nutrition b. Avoiding alcohol ingestion c. Taking lactulose (Cephulac) d. Using vitamin B supplements

ANS: C The risk of contracting a primary carcinoma of the liver is higher in clients with cirrhosis from any cause. Blunt liver trauma, diabetes mellitus, and chronic malnutrition do not increase a person's risk for developing liver cancer.

A nurse assesses clients on the medical-surgical unit. Which client is at greatest risk for the development of carcinoma of the liver? a. A 22-year-old with a history of blunt liver trauma b. A 48-year-old with a history of diabetes mellitus c. A 66-year-old who has a history of cirrhosis d. An 82-year-old who has chronic malnutrition

ANS: B The route of acquisition of hepatitis A infection is through close personal contact or ingestion of contaminated water or shellfish. Hepatitis A is not transmitted through medications, blood transfusions, or Epstein-Barr virus. Toxic and drug-induced hepatitis is caused from exposure to hepatotoxins, but this is not a form of hepatitis A. Hepatitis B can be spread through blood transfusions. Epstein-Barr virus causes a secondary infection that is not associated with hepatitis A.

After teaching a client who has been diagnosed with hepatitis A, the nurse assesses the client's understanding. Which statement by the client indicates a correct understanding of the teaching? a. "Some medications have been known to cause hepatitis A." b. "I may have been exposed when we ate shrimp last weekend." c. "I was infected with hepatitis A through a recent blood transfusion." d. "My infection with Epstein-Barr virus can co-infect me with hepatitis A."

Based on the degree of liver involvement and decreased function, protein intake may have to be decreased. However, some protein is necessary for the synthesis of albumin and normal healing. The other statements indicate accurate understanding of self-care measures for this client

After teaching a client who has alcohol-induced cirrhosis, a nurse assesses the client's understanding. Which statement made by the client indicates a need for additional teaching? a. "I cannot drink any alcohol at all anymore." b. "I need to avoid protein in my diet." c. "I should not take over-the-counter medications." d. "I should eat small, frequent, balanced meals."

ANS: C Mutations in both BRCA2 and p16 genes increase the risk for developing pancreatic cancer in a small number of cases. The other factors do not appear to be linked to increased risk.

The nurse is teaching a community group about pancreatic cancer. Which risk factor does the nurse instruct is known for development of this type of cancer? a. Hypothyroidism b. Cholelithiasis c. BRCA2 gene mutation d. African-American ethnicity

ANS: A Using the ABCs, airway and oxygenation status should always be assessed first. Next, the nurse should assess the IV line (circulation). After that, the other two items can be assessed.

The postanesthesia care unit nurse is caring for a client who has just undergone an open Whipple procedure. The client has multiple tubes and drains in place after the surgery. Which does the nurse assess first? a. Endotracheal tube with 40% fraction of inspired oxygen (FiO2) b. Foley catheter to bedside drainage c. Nasogastric tube to low intermittent suction d. Triple-lumen IV catheter with lactated Ringer's solution

Slowed growth The growth of children on long-term inhaled steroids should be assessed frequently to assess for systemic effects of these drugs. Cough is prevented by inhaled steroids. No evidence exists that inhaled steroids cause osteoporosis. Cushing's syndrome is caused by long-term systemic steroids.

It is now recommended that children with asthma who are taking long-term inhaled steroids should be assessed frequently to monitor for what increased risk?

ANS: A The client who has undergone a laparoscopic cholecystectomy may report free air pain because of retention of carbon dioxide in the abdomen. The nurse assists the client with early ambulation to promote absorption of the carbon dioxide. Coughing and deep breathing are important postoperative activities, but they are not related to discomfort from carbon dioxide. Cold compresses and drinking tea would not be helpful.

The nurse is caring for a postoperative client who reports pain in the shoulder blades following laparoscopic cholecystectomy surgery. Which direction does the nurse give to the nursing assistant to help relieve the client's pain? a. "Ambulate the client in the hallway." b. "Apply a cold compress to the client's back." c. "Encourage the client to take sips of hot tea or broth." d. "Remind the client to cough and deep breathe every hour.

ANS: B After cholecystectomy, clients need a nutritious diet without a lot of excess fat; otherwise a special diet is not recommended for most clients. Good fluid intake is healthy for all people but is not related to the surgery. Drinking fluids between meals helps with dumping syndrome, which is not seen with this operation. Restriction of sweets is not required.

The nurse is providing discharge teaching for a client who has just undergone laparoscopic cholecystectomy surgery. Which statement by the client indicates understanding of the instructions? a. "I will drink at least 2 liters of fluid a day." b. "I need a diet without a lot of fatty foods." c. "I should drink fluids between meals rather than with meals." d. "I will avoid concentrated sweets and simple carbohydrates."

ANS: D The provider will discontinue the T-tube. The other statements are accurate.

The nurse is providing discharge teaching for a client who will be going home with a T-tube following cholecystectomy surgery. Which statement by the client indicates the need for additional teaching? a. "I will keep the drainage bag lower than the tube itself." b. "I will inspect the T-tube drainage site daily for signs of infection." c. "I will be careful not to pull on the tube or to accidentally pull it out." d. "I will slowly pull about an inch of the tube out each day until it's out."

ANS: C The enzymes should be taken immediately before eating meals or snacks. The client should wipe his or her lips carefully after drinking the enzyme preparation because the liquid could damage the skin. If the client cannot swallow the capsules whole, they can be opened up and the powder sprinkled on applesauce, mashed fruit, or rice cereal. Protein items will be dissolved by the enzymes if they are mixed together.

The nurse is providing discharge teaching for a client who will be receiving pancreatic enzyme replacement at home. Which statement by the client indicates that additional teaching is needed? a. "The capsules can be opened and the powder sprinkled on applesauce if needed." b. "I will wipe my lips carefully after I drink the enzyme preparation." c. "The best time to take the enzymes is immediately after I have a meal or a snack." d. "I will not mix the enzyme powder with food or liquids that contain protein."

ANS: D Clients with cholelithiasis should avoid foods high in fat and cholesterol, such as whole milk, butter, and fried foods. Lasagna, 2% milk, grilled cheese, cream, and butter all have high levels of fat. The meal with the least amount of fat is the chicken breast dinner.

The nurse is teaching a client with a history of cholelithiasis to select menu items for dinner. Which selections made by the client indicate that the nurse's teaching was effective? a. Lasagna, tossed salad with Italian dressing, 2% milk b. Grilled cheese sandwich, tomato soup, coffee with cream c. Caesar salad with chicken, soft breadstick with butter, diet cola d. Roasted chicken breast, baked potato with chives, hot tea with sugar

A nurse cares for a client who is recovering from laparoscopic cholecystectomy surgery. The client reports pain in the shoulder blades. How should the nurse respond? a. "Ambulating in the hallway twice a day will help." b. "I will apply a cold compress to the painful area on your back." c. "Drinking a warm beverage can relieve this referred pain." d. "You should cough and deep breathe every hour."

ANS: A The client who has undergone a laparoscopic cholecystectomy may report free air pain due to retention of carbon dioxide in the abdomen. The nurse assists the client with early ambulation to promote absorption of the carbon dioxide. Cold compresses and drinking a warm beverage would not be helpful. Coughing and deep breathing are important postoperative activities, but they are not related to discomfort from carbon dioxide.

After teaching a client who has a history of cholelithiasis, the nurse assesses the client's understanding. Which menu selection made by the client indicates the client clearly understands the dietary teaching? a. Lasagna, tossed salad with Italian dressing, and low-fat milk b. Grilled cheese sandwich, tomato soup, and coffee with cream c. Cream of potato soup, Caesar salad with chicken, and a diet cola d. Roasted chicken breast, baked potato with chives, and orange juice

ANS: D Clients with cholelithiasis should avoid foods high in fat and cholesterol, such as whole milk, butter, and fried foods. Lasagna, low-fat milk, grilled cheese, cream, and cream of potato soup all have high levels of fat. The meal with the least amount of fat is the chicken breast dinner.

ANS: B The protein in the blood in the gastrointestinal (GI) tract will be absorbed and may result in an increase in the ammonia level because the liver cannot metabolize protein very well.

14. To detect possible complications in a patient with severe cirrhosis who has bleeding esophageal varices, it is most important for the nurse to monitor a. bilirubin levels. b. ammonia levels. c. potassium levels. d. prothrombin time.

ANS: B Spironolactone is a potassium-sparing diuretic and will help increase the patient's potassium level. The nurse does not need to talk with the doctor before giving the spironolactone, although the health care provider should be notified about the low potassium value. The furosemide will further decrease the patient's potassium level and should be held until the nurse talks with the health care provider.

10. A serum potassium level of 3.2 mEq/L (3.2 mmol/L) is reported for a patient with cirrhosis who has scheduled doses of spironolactone (Aldactone) and furosemide (Lasix). due. Which action should the nurse take? b. Administer the spironolactone.

ANS: B Extending the arms allows the nurse to check for asterixis, a classic sign of hepatic encephalopathy. The other tests might also be done as part of the neurologic assessment but would not be diagnostic for hepatic encephalopathy.

11. Which action should the nurse take to evaluate treatment effectiveness for a patient who has hepatic encephalopathy? b. Ask the patient to extend both arms forward.

ANS: C Amylase is elevated in acute pancreatitis. Although changes in the other values may occur, they would not be useful in evaluating whether the prescribed therapies have been effective.

19. Which laboratory test result will the nurse monitor when evaluating the effects of therapy for a 62-year-old female patient who has acute pancreatitis? a. Calcium b. Bilirubin c. Amylase d. Potassium

"Most people with hypertension do not have symptoms." Most people with hypertension are asymptomatic, although a small percentage do have symptoms such as headache. The nurse should explain this to the client. Asking about paying for medications is not related because the client has already admitted nonadherence. Threatening the client with possible complications will not increase compliance.

A client has been diagnosed with hypertension but does not take the antihypertensive medications because of a lack of symptoms. What response by the nurse is best?

Ask the spouse to explain the fear of visiting in further detail. The nurse needs to obtain further information about the spouse's specific fears so they can be addressed. This will decrease stress and permit visitation, which will be beneficial for both client and spouse. Precautions for TB prevent transmission to all who come into contact with the client. Explaining isolation precautions and what to do when entering the room will be helpful, but this is too narrow in scope to be the best answer. Telling the spouse it's safe to visit is demeaning of the spouse's feelings.

A client has been hospitalized with tuberculosis (TB). The client's spouse is fearful of entering the room where the client is in isolation and refuses to visit. What action by the nurse is best?

3. Ranitidine blocks secretion of hydrochloric acid. Clients who take only one daily dose of ranitidine are usually advised to take it at bedtime to inhibit nocturnal secretion of acid. Clients who take the drug twice a day are advised to take it in the morning and at bedtime. It is not necessary to take the drug before meals. The client should take the drug regularly, not just when pain occurs.

A client is to take one daily dose of ranitidine (Zantac) at home to treat her peptic ulcer. The client understands proper drug administration of ranitidine when she says that she will take the drug at which of the following times? 1. Before meals. 2. With meals. 3. At bedtime. 4. When pain occurs.

Arrange a home safety evaluation.

A client with osteoporosis is going home, where the client lives alone. What action by the nurse is best?

Severe, steady right lower quadrant pain Right lower quadrant pain, specifically at McBurney's point, is characteristic of appendicitis. Usually if nausea and vomiting begin first, the client has gastroenteritis. Marked peristalsis and hyperactive bowel sounds are not indicative of appendicitis. Abdominal pain due to appendicitis decreases with knee flexion.

A nurse assesses a client who has appendicitis. Which clinical manifestation should the nurse expect to find?

ANS: A - Right lower quadrant pain, specifically at McBurneys point, is characteristic of appendicitis. Usually if nausea and vomiting begin first, the client has gastroenteritis. Marked peristalsis and hyperactive bowel sounds are not indicative of appendicitis. Abdominal pain due to appendicitis decreases with knee flexion.

A nurse assesses a client who has appendicitis. Which clinical manifestation should the nurse expect to find? a. Severe, steady right lower quadrant pain b. Abdominal pain associated with nausea and vomiting c. Marked peristalsis and hyperactive bowel sounds d. Abdominal pain that increases with knee flexion

ANS: A, D, F Obesity, pregnancy, and diabetes are all risk factors for the development of cholelithiasis. A diet low in saturated fats and moderate alcohol intake may decrease the risk. Although metabolic syndrome is a precursor to diabetes, it is not a risk factor for cholelithiasis. The client should be informed of the connection.

A nurse assesses a client with cholelithiasis. Which assessment findings should the nurse identify as contributors to this client's condition? (Select all that apply.) a. Body mass index of 46 b. Vegetarian diet c. Drinking 4 ounces of red wine nightly d. Pregnant with twins e. History of metabolic syndrome f. Glycosylated hemoglobin level of 15%

Slurred speech and confusion A left-sided cardiac catheterization specifically increases the risk for a cerebral vascular accident. A change in neurologic status needs to be acted on immediately. Discomfort and bruising are expected at the site. If intake decreases, a patient can become dehydrated because of dye excretion. The second intervention would be to increase the patient's fluid status. Neurologic changes would take priority.

A nurse assesses a patient who is recovering after a left-sided cardiac catheterization. Which assessment finding requires immediate intervention?

Contact the provider and prepare for intubation. Stridor on exhalation is a hallmark of respiratory distress, usually caused by obstruction resulting from edema. The nurse should prepare to assist with emergency intubation or tracheostomy while notifying the provider or the Rapid Response Team. Stridor is an emergency situation; therefore, reassuring the patient, documenting, and reassessing in an hour do not address the urgency of the situation. Oxygen should be applied, but this action will not keep the airway open.

A nurse assesses a patient who is recovering from a total thyroidectomy and notes the development of stridor. What action does the nurse take first?

ans:A Hepatitis B can be spread through sexual contact, needle sharing, needle sticks, blood transfusions, hemodialysis, acupuncture, and the maternal-fetal route. A person with multiple sexual partners has more opportunities to contract the infection. Hepatitis B is not transmitted through medications, casual contact with other travelers, or raw shellfish. Although an overdose of acetaminophen can cause liver cirrhosis, this is not associated with hepatitis B. Hepatitis E is found most frequently in international travelers. Hepatitis A is spread through ingestion of contaminated shellfish.

A nurse assesses clients at a community health fair. Which client is at greatest risk for the development of hepatitis B? a. A 20-year-old college student who has had several sexual partners b. A 46-year-old woman who takes acetaminophen daily for headaches c. A 63-year-old businessman who travels frequently across the country d. An 82-year-old woman who recently ate raw shellfish for dinner

ANS: D Risk factors for pancreatic cancer include obesity, older age, high intake of red meat, and cigarette smoking. Sushi and wine intake are not risk factors for pancreatic cancer.

A nurse assesses clients on the medical-surgical unit. Which client should the nurse identify as at high risk for pancreatic cancer? a. A 26-year-old with a body mass index of 21 b. A 33-year-old who frequently eats sushi c. A 48-year-old who often drinks wine d. A 66-year-old who smokes cigarettes

ANS: B A low-sodium diet is one means of controlling abdominal fluid collection. Monitoring intake and output does not control fluid accumulation, nor does weighing the client. These interventions merely assess or monitor the situation. Increasing fluid intake would not be helpful.

A nurse cares for a client who has cirrhosis of the liver. Which action should the nurse take to decrease the presence of ascites? a. Monitor intake and output. b. Provide a low-sodium diet. c. Increase oral fluid intake. d. Weigh the client daily.

Increased proteins Increased carbohydrates Increased calorie intake

A nurse teaches a patient with hyperthyroidism. Which dietary modifications should the nurse include in this patient's teaching? (Select all that apply.)

ANS: B For safety, the client should void just before a paracentesis. The nurse or the provider should have the client sign the consent form. The proper position for a paracentesis is sitting upright in bed or, alternatively, sitting on the side of the bed and leaning over the bedside table. The client will be on bedrest after the procedure.

A nurse cares for a client who is scheduled for a paracentesis. Which intervention should the nurse delegate to an unlicensed assistive personnel (UAP)? a. Have the client sign the informed consent form. b. Assist the client to void before the procedure. c. Help the client lie flat in bed on the right side. d. Get the client into a chair after the procedure.

ANS: A, B Clients who are experiencing biliary colic may present with tachycardia, pallor, diaphoresis, prostration, or other signs of shock. The nurse should stay with the client, lower the client's head, and contact the provider or Rapid Response Team for immediate assistance. Treatment for shock usually includes intravenous fluids; therefore, decreasing fluids would be an incorrect intervention. The client's tachycardia is a result of shock, not pain. Performing the vagal maneuver or administering opioids could knock out the client's compensation mechanism.

A nurse cares for a client who presents with tachycardia and prostration related to biliary colic. Which actions should the nurse take? (Select all that apply.) a. Contact the provider immediately. b. Lower the head of the bed. c. Decrease intravenous fluids. d. Ask the client to bear down. e. Administer prescribed opioids.

-Performs hand hygiene and positions the client in high-Fowlers position, with pillows behind the head and shoulders -Checks for correct placement by checking the pH of the fluid aspirated from the tube -Connects the NG tube to intermittent medium suction with an anti-reflux valve on the air vent

A nurse inserts a nasogastric (NG) tube for an adult client who has a bowel obstruction. Which actions does the nurse perform correctly? (Select all that apply.)

Pressure ulcer A pressure ulcer does not have approximated edges. Pressure ulcers usually involve the epidermis and dermis that requires healing by secondary intention. Healing takes place from inner to outer.

A nurse is assessing several wounds. Which type of wound should the nurse anticipate will heal by secondary intention?

B A client with botulism is at risk for respiratory failure. This client's respiratory rate is slow and shallow, which could indicate impending respiratory distress or failure. The nurse should remain with the client while another nurse notifies the provider. Nothing is allowed by mouth until all respiratory function and swallowing are normal. The nurse should monitor and document the IV infusion per protocol, but this does not take priority. Allowing the client to rest and ordering food items are not appropriate actions.

A nurse is caring for a client hospitalized with botulism. The nurse obtains the following vital signs: temperature—99.8° F (37.6° C), pulse—100, respiratory rate—10 and shallow, and blood pressure—100/62 mm Hg. What action by the nurse is most appropriate? a. Allow the client rest periods without interruption. b. Stay with the client while another nurse calls the physician. c. Check the client's IV rate and document all findings. d. Help the client order appropriate food items from the menu.

exercise for 30 minutes daily preferably after a meals

A nurse is teaching a client who has been newly diagnosed with type 2 DM. Which teaching point should the nurse emphasize?

A, D Washing hands after contact with animals and using separate cutting boards for meat and other foods will help prevent E. coli infection. The other statements are not related to preventing E. coli infection.

A nurse is teaching a community group ways to prevent Escherichia coli infection. Which statements made by the nurse are accurate? (Select all that apply.) a. "Wash your hands after any contact with animals." b. "It is not necessary to buy a meat thermometer." c. "Stay away from people who are ill with diarrhea." d. "Use separate cutting boards for meat and vegetables." e. "Avoid swimming in backyard pools and using hot tubs."

Fluid Ascites is free fluid in the peritoneal cavity and occurs with heart failure, portal hypertension, cirrhosis, hepatitis, pancreatitis, and cancer.

A nurse notices that a patient has ascites, which indicates the presence of:

Maintain nothing by mouth (NPO) and administer intravenous fluids. The client should be kept NPO to reduce GI activity and reduce pancreatic enzyme production. IV fluids should be used to prevent dehydration. The client may need a nasogastric tube. Pain medications should be given around the clock and more frequently than every 4 to 6 hours. A fetal position with legs drawn up to the chest will promote comfort.

A nurse plans care for a client with acute pancreatitis. Which intervention should the nurse include in this client's plan of care to reduce discomfort?

Assistance with activities of daily living A patient with dyspnea and difficulty completing activities such as climbing a flight of stairs has class III dyspnea. The nurse would provide assistance with activities of daily living. These patients would be encouraged to participate in activities as tolerated. They would not be on complete bedrest, may not be able to tolerate daily physical therapy, and only need oxygen if hypoxia is present.

A nurse plans care for a patient who is experiencing dyspnea and must stop multiple times when climbing a flight of stairs. Which intervention would the nurse include in this patient's plan of care?

ANS: A - Fistulas place the client with Crohns disease at risk for hypokalemia which can lead to serious dysrhythmias. This potassium level is low and should cause the nurse to intervene. The white blood cell count is normal. The other two findings are abnormal and also warrant intervention, but the potassium level takes priority.

A nurse reviews the chart of a client who has Crohns disease and a draining fistula. Which documentation should alert the nurse to urgently contact the provider for additional prescriptions? a. Serum potassium of 2.6 mEq/L b. Client ate 20% of breakfast meal c. White blood cell count of 8200/mm3 d. Clients weight decreased by 3 pounds

45 mL

A nurse teaches a client experiencing heartburn to take 1 ½ oz of Maalox when symptoms appear. How many milliliters should the client take? ________________________ mL. .

Public Public communication is interaction with an audience. Nurses have opportunities to speak with groups of consumers about health-related topics, present scholarly work to colleagues at conferences, or lead classroom discussions with peers or students. When nurses work on committees or participate in patient care conferences, they use a small group communication process. Interpersonal communication is one-on-one interaction between a nurse and another person that often occurs face to face. Intrapersonal communication is a powerful form of communication that you use as a professional nurse. This level of communication is also called self-talk.

A nurse wants to present information about flu immunizations to the older adults in the community. Which type of communication should the nurse use?

Have the patient help mark the surgical site. Ensure that the patient has an armband on. Allow the patient to use the toilet before giving sedation. Check that consent is on the chart.

A nurse working in the preoperative holding area performs which functions to ensure patient safety? (Select all that apply.)

"I will decrease the amount of fatty foods in my diet." After cholecystectomy, clients need a nutritious diet without a lot of excess fat; otherwise a special diet is not recommended for most clients. Good fluid intake is healthy for all people but is not related to the surgery. Drinking fluids between meals helps with dumping syndrome, which is not seen with this procedure. Restriction of sweets is not required.

After teaching a client who is recovering from laparoscopic cholecystectomy surgery, the nurse assesses the client's understanding. Which statement made by the client indicates a correct understanding of the teaching?

ANS: A, C, D - Taking warm sitz baths each day, using bulk-producing agents, and administering anti-inflammatory suppositories are all appropriate actions for the client with an anal fissure. The client should not use enemas or laxatives to promote elimination, but rather should rely on bulk-producing agents such as psyllium hydrophilic mucilloid (Metamucil).

After teaching a client with an anal fissure, a nurse assesses the clients understanding. Which client actions indicate that the client correctly understands the teaching? (Select all that apply.) a. Taking a warm sitz bath several times each day b. Utilizing a daily enema to prevent constipation c. Using bulk-producing agents to aid elimination d. Self-administering anti-inflammatory suppositories e. Taking a laxative each morning

He may have trouble chewing, so I will offer bite-sized portions.

After teaching the wife of a client who has Parkinson disease, the nurse assesses the wifes understanding. Which statement by the clients wife indicates she correctly understands changes associated with this disease?

"Pain control is a major component of the care provided by hospice and its staff members." Correct: This response correctly describes the services provided by hospice and its staff members, and helps reassure the client about their expertise in pain management.

The client has been diagnosed with terminal gastric cancer and is interested in obtaining support from hospice, but expresses concern that pain management will not be adequate. What is the nurse's best response?

Joints

The functional units of the musculoskeletal system are the:

B This client may have appendicitis based on RLQ pain. A white blood cell count of 22,000/mm3 is severely elevated and could indicate a perforated appendix, as could the fever. The nurse should bring these findings to the provider's attention as soon as possible. A left shift would be expected in uncomplicated appendicitis. The sodium reading is only slightly high; this could be due to hemoconcentration from vomiting or from decreased intake. The creatinine level is normal.

The nurse conducts a physical assessment for a client with severe right lower quadrant (RLQ) abdominal pain. The nurse notes that the abdomen is rigid and the client's temperature is 101.1° F (38.4° C). Which laboratory value does the nurse bring to the attention of the health care provider as a priority? a. A "left shift" in the white blood cell count b. White blood cell count, 22,000/mm3 c. Serum sodium, 149 mEq/L d. Serum creatinine, 0.7 mg/dL

run normal saline at a KVO rate

The nurse determines that a client is having a transfusion reaction. After the nurse stops the transfusion, which action should immediately be taken?

B Effluent from an ileostomy will become less liquid (but not solid) over time as the body adapts to loss of the large bowel. This process takes time and the client should be reassured of this. Clients with a new ileostomy should avoid high-fiber diets for the first few weeks because blockage of the bowel may occur. Buttermilk, yogurt, and carbonated drinks will not affect this process.

The nurse is caring for a client who had ileostomy surgery 10 days ago. The client verbalizes concerns that the effluent has not become formed and is still liquid green. Which is the nurse's best response? a. "I will call your health care provider right away because the stool should be semi-solid by now." b. "Your stools will firm up in a few weeks as your body gets used to the ileostomy." c. "You should eat a high-fiber diet to help make the stool bulkier and more solid." d. "You can add buttermilk or yogurt to your diet and avoid carbonated soft drinks."

A The client should drink plenty of fluids to prevent dehydration. Clients are not necessarily restricted to clear liquids. Milk products may not be tolerated. Caffeinated beverages increase intestinal motility and should be avoided.

The nurse is caring for a client who has acute viral gastroenteritis. Which dietary instruction does the nurse provide to the client? a. "Drink plenty of fluids to prevent dehydration." b. "You can have only clear liquids to drink." c. "Milk products will give you extra protein." d. "You can have sips of cola or tea to relieve nausea."

protect the trachea from trauma as the tube is inserted

The purpose of the obturator in the tracheostomy set is:

D The level of serum transaminase, a liver enzyme, is elevated with liver trauma. The other laboratory values are within normal limits and are not specific for the liver.

The nurse is caring for a client who has suffered abdominal trauma in a motor vehicle crash. Which laboratory finding indicates that the client's liver was injured? a. Serum lipase, 49 U/L b. Serum amylase, 68 IU/L c. Serum creatinine, 0.8 mg/dL d. Serum transaminase, 129 IU/L

A Once a person has developed a polyp, risk for occurrence of multiple polyps is present. The physician usually can remove all visible polyps during the colonoscopy procedure. Follow-up colonoscopy is not done to ensure that healing occurred where a polyp was removed, or to check for cancerous lesions that were not visible during the first procedure.

The nurse is caring for a client who has undergone removal of a benign colonic polyp. The client asks the nurse why a follow-up colonoscopy is necessary. Which is the nurse's best response? a. "You are at risk for developing more polyps in the future." b. "You may have other cancerous lesions that could not be seen right now." c. "The doctor can remove only a few of the polyps during each colonoscopy." d. "This test will ensure that you have healed where the polyp was removed."

C The client is instructed to avoid meat, aspirin, vitamin C, and anti-inflammatory drugs for 48 hours before the test. The other directions are not accurate for this test.

The nurse is caring for a client who is scheduled to have fecal occult blood testing. Which instructions does the nurse give to the client? a. "You must fast for 12 hours before the test." b. "You will be given a cleansing enema the morning of the test." c. "You must avoid eating meat for 48 hours before the test." d. "You will be sedated and will require someone to accompany you home."

D Asacol is enteric coated and should not be crushed, chewed, or broken. If the client is unable to swallow the Asacol pill, Rowasa enemas may be administered instead, with a provider's order. Asacol is not available as a suspension or elixir.

The nurse is caring for a client who is taking mesalamine (5-aminosalicylic acid) (Asacol, Rowasa) for ulcerative colitis. The client has trouble swallowing the pill. Which action by the nurse is most appropriate? a. Crush the pill carefully and administer it to the client in applesauce or pudding. b. Empty the contents of the capsule into applesauce or pudding for administration. c. Contact the client's health care provider to request an order for Asacol suspension. d. Contact the client's health care provider to request an order for Rowasa enemas instead.

A The client with a hernia presenting with abdominal pain, fever, tachycardia, nausea and vomiting, and hypoactive bowel sounds should be suspected of having developed strangulation. Strangulation poses a risk of intestinal obstruction. The client should be placed on NPO status, and the health care provider should be notified. The symptoms are not suggestive of enlargement of the hernia, adhesion formation, or bowel perforation.

The nurse is caring for a client with an umbilical hernia who reports increased abdominal pain, nausea, and vomiting. The nurse notes high-pitched bowel sounds. Which conclusion does the nurse draw from these assessment findings? a. Bowel obstruction; client should be placed on NPO status. b. Perforation of the bowel; client needs emergency surgery. c. Adhesions in the hernia; client needs elective surgery. d. Hernia is dangerously enlarged; client needs a nasogastric (NG) tube.

B Toilet paper can irritate the sensitive perineal skin, so warm water rinses or soft cotton washcloths should be used instead. Although aloe vera may facilitate healing of superficial abrasions, it is not an effective skin barrier for diarrhea. Skin barriers such as zinc oxide and vitamin A and D ointment help protect the rectal area from the excoriating effects of liquid stools. Patting the skin is recommended instead of rubbing the skin dry.

The nurse is caring for a client with perineal excoriation caused by diarrhea from acute gastroenteritis. Which client statement indicates that additional teaching about perineal care is needed? a. "I will rinse my rectal area with warm water after each stool and then apply zinc oxide ointment." b. "I will clean my rectal area thoroughly with toilet paper after each stool and then apply aloe vera gel." c. "I will take a sitz bath three times a day and then pat my rectal area gently but thoroughly to make sure I am dry." d. "I will clean my rectal area with a soft cotton washcloth and then apply vitamin A and D ointment."

C Although the client with severe diarrhea may experience skin irritation and hypovolemia, the client is most at risk for cardiac dysrhythmias secondary to potassium and magnesium loss from severe diarrhea. The client should have her or his electrolyte levels monitored, and electrolyte replacement may be necessary. Abdominal percussion is an important part of physical assessment but has lower priority for this client than heart rate and rhythm.

The nurse is caring for a client with ulcerative colitis and severe diarrhea. Which nursing assessment is the highest priority? a. Skin integrity b. Blood pressure c. Heart rate and rhythm d. Abdominal percussion

Blood pressure is 90/40 mm Hg. The blood pressure indicates that the patient may be developing hypovolemic shock as a result of intravascular fluid loss due to the burn injury. This finding will require immediate intervention to prevent the complications associated with systemic hypoperfusion. The poor oral intake, decreased urine output, and skin tenting all indicate the need for increasing the patient's fluid intake but not as urgently as the hypotension.

The nurse is caring for a patient with a massive burn injury and possible hypovolemia. Which assessment data will be of most concern to the nurse?

pH 7.32, PaCO2 38, HCO3 20

Which ABG value indicates uncompensated metabolic acidosis?

ANS: A Genotyping of HCV has an important role in managing treatment and is done before drug therapy is initiated.

3. A 36-year-old male patient in the outpatient clinic is diagnosed with acute hepatitis C (HCV) infection. Which action by the nurse is appropriate? a. Schedule the patient for HCV genotype testing.

ANS: C The client should be advised to avoid fresh, raw fruits and vegetables because they can be contaminated by tap water. Drinking bottled water, and not sharing plates, glasses, or eating utensils are good ways to prevent illness, as is careful handwashing.

After teaching a client who has plans to travel to a non-industrialized country, the nurse assesses the client's understanding regarding the prevention of viral hepatitis. Which statement made by the client indicates a need for additional teaching? a. "I should drink bottled water during my travels." b. "I will not eat off another's plate or share utensils." c. "I should eat plenty of fresh fruits and vegetables." d. "I will wash my hands frequently and thoroughly."

ANS: B, D Symptoms of hemorrhage and hypovolemic shock include hypotension, tachycardia, tachypnea, pallor, diaphoresis, cool and clammy skin, and confusion

An emergency room nurse assesses a client with potential liver trauma. Which clinical manifestations should alert the nurse to internal bleeding and hypovolemic shock? (Select all that apply.) a. Hypertension b. Tachycardia c. Flushed skin d. Confusion e. Shallow respirations

ANS: A, C, D, E Nurses should always use Standard Precautions for client care, and policies should reflect this. Needleless systems and reduction of sharps can help prevent hepatitis. Postexposure prophylaxis should be provided immediately. All health care workers should receive the hepatitis vaccinations that are available.

An infection control nurse develops a plan to decrease the number of health care professionals who contract viral hepatitis at work. Which ideas should the nurse include in this plan? (Select all that apply.) a. Policies related to consistent use of Standard Precautions b. Hepatitis vaccination mandate for workers in high-risk areas c. Implementation of a needleless system for intravenous therapy d. Number of sharps used in client care reduced where possible e. Postexposure prophylaxis provided in a timely manner

Hypercarbia

The patient has been on oxygen therapy at 70% for over 48hours. For which complication must the nurse monitor?

The onset of hepatitis B is insidious. Hepatitis B can be prevented by hepatitis B virus vaccine. Immunity to hepatitis B occurs after one attack. Hepatitis B can be transferred to an infant of a breastfeeding mother.

Which statements regarding hepatitis B are correct? (Select all that apply.)

ANS: D NG suction and NPO status will decrease the release of pancreatic enzymes into the pancreas and decrease pain.

28. A 67-year-old male patient with acute pancreatitis has a nasogastric (NG) tube to suction and is NPO. Which information obtained by the nurse indicates that these therapies have been effective? c. Electrolyte levels are normal. d. Abdominal pain is decreased.

Swimming Swimming is well tolerated in children with EIB because they are breathing air fully saturated with moisture and because of the type of breathing required in swimming. Exercise-induced bronchospasm is more common in sports that involve endurance, such as soccer, running, and basketball. Prophylaxis with medications may be necessary.

A parent whose two school-age children diagnosed with exercise-induced bronchospasm (EIB) asks the nurse in what sports, if any, they can participate. The nurse should recommend which sport?

Apply a pulse ox and cardiac monitor

A patient come to the ED extremely anxious, tachycardiac, struggling for air with a moist cough that is productive and blood tinged. What is the priority nursing action?

Starting a large-bore intravenous (IV) Correct: A large-bore IV should be placed as requested, so that blood products can be administered.

The client is experiencing bleeding related to peptic ulcer disease (PUD). Which nursing intervention is the highest priority?

parathyroid

A disorder of which organ can result in changes in serum calcium levels:

ANS: B - Ciprofloxacin should be taken for 10 to 14 days to treat Salmonella infection, and should not be stopped once the diarrhea has cleared. Clients should be advised to take the entire course of medication. People with Salmonella should not prepare foods for others because the infection may be spread in this way. Hands should be washed with antibacterial soap before and after eating to prevent spread of the bacteria. Dishes and eating utensils should not be shared and should be cleaned thoroughly. Clients can be carriers for up to 1 year.

After teaching a client who was hospitalized for Salmonella food poisoning, a nurse assesses the clients understanding. Which statement made by the client indicates a need for additional teaching? a. I will let my husband do all of the cooking for my family. b. Ill take the ciprofloxacin until the diarrhea has resolved. c. I should wash my hands with antibacterial soap before each meal. d. I must place my dishes into the dishwasher after each meal.

a new tissue is formed that is the same as the damaged tissue

When a wound heals through regeneration:

My child should not eat bananas or kiwis.

The nurse is talking to a parent with a child who has a latex allergy. Which statement by the parent would indicate a correct understanding of the teaching?

Folic acid for all women of childbearing age.

A current recommendation to prevent neural tube defects is the supplementation of:

Neurogenic bladder.

The most common problem of children born with a myelomeningocele is:

-Diagnosed with Cushing's disease -Has a thyroid disorder

The nurse knows that which patients are at risk of developing secondary osteoporosis? (Select all that apply.)

Practice proper handwashing

The nurse is teaching incisional care to a patient who is being discharged after abdominal surgery. Which priority instruction must the nurse include?

alkalosis

The nurse would analyze an arterial blood pH of 7.48 as indicating:

ANS: B The most common complication of balloon tamponade is aspiration pneumonia. In addition, if the gastric balloon ruptures, the esophageal balloon may slip upward and occlude the airway.

13. A 53-year-old patient is being treated for bleeding esophageal varices with balloon tamponade. Which nursing action will be included in the plan of care? b. Monitor the patient for shortness of breath.

Light-colored stool Jaundice, clay-colored stools, and dark urine are more commonly seen with chronic cholecystitis. The other symptoms are seen equally with both chronic and acute cholecystitis.

A nurse assesses a client who has cholecystitis. Which clinical manifestation indicates that the condition is chronic rather than acute?

ANS: B Fever, right quadrant or flank pain, and jaundice are signs of liver transplant rejection; the client should be admitted to the hospital as soon as possible for intervention. Anti-rejection drugs do make a client more susceptible to infection, but this client has signs of rejection, not infection. The nurse should not advise the client to take an additional dose of cyclosporine or acetaminophen as these medications will not treat the acute rejection.

A telehealth nurse speaks with a client who is recovering from a liver transplant 2 weeks ago. The client states, "I am experiencing right flank pain and have a temperature of 101° F." How should the nurse respond? a. "The anti-rejection drugs you are taking make you susceptible to infection." b. "You should go to the hospital immediately to have your new liver checked out." c. "You should take an additional dose of cyclosporine today." d. "Take acetaminophen (Tylenol) every 4 hours until you feel better."

A 50-year-old who has the BRCA2 gene mutation Mutations in both the BRCA2 and p16 genes increase the risk for developing pancreatic cancer in a small number of cases. The other factors do not appear to be linked to increased risk.

A nurse assesses clients at a community health center. Which client is at highest risk for pancreatic cancer?

First and second heart sounds

A patient's chart notes that the examiner has heard S1 and S2 on auscultation of the heart. What does this documentation refer to?

Please come to the clinic today to have your arm checked by the provider.

A phone triage nurse speaks with a client who has an arm cast. The client states, My arm feels really tight and puffy. How should the nurse respond?

A 52-year-old in a tripod position using accessory muscles to breathe The client who is in a tripod position and using accessory muscles is working to breathe. This client must be assessed first to establish how well the client is breathing and provide interventions to minimize respiratory failure. The other clients are not in acute distress.

A pulmonary nurse cares for clients who have chronic obstructive pulmonary disease (COPD). Which client should the nurse assess first?

Palpating both carotid arteries at the same time The student should not compress both carotid arteries at the same time to avoid brain ischemia. Blood pressure should be taken and compared in both arms. Prolonged capillary refill is considered to be greater than 5 seconds in an older adult, so classifying refill of 4 seconds as normal would not require intervention. Bruits should be auscultated.

A student nurse is assessing the peripheral vascular system of an older adult. What action by the student would cause the faculty member to intervene?

"If you are concerned that you are at high risk to develop gastric cancer, I would recommend that you speak to your physician about the possibility of genetic testing." Correct: Genetic counseling will help the client determine whether he is at exceptionally high risk to develop gastric cancer.

The client with peptic ulcer disease (PUD) asks the nurse whether a maternal history of ovarian cancer will cause the client to develop gastric cancer. What is the nurse's best response?

"These herbs could be helpful. However, you should talk with your physician before adding them to your treatment regimen." Correct: Although these herbs may be helpful in managing PUD, the client should consult his or her physician before making a change in the treatment regimen.

The client with peptic ulcer disease (PUD) asks the nurse whether licorice and slippery elm might be useful in managing the disease. What is the nurse's best response?

Chicken and rice Correct: Chicken and rice is the only selection suitable for the client who is experiencing dumping syndrome because it contains high protein without the addition of milk or wheat products.

The nurse and the dietitian are planning sample diet menus for the client who is experiencing dumping syndrome. Which sample meal is best for this client?

Stage II The ulcer is nonblanchable erythematous that is intact is Stage I. Stage II is reddened by the skin also broken, giving the appearance of a broken blister

The nurse assessing a bedridden client notes a large erythemic area on the client's buttocks. In addition, the center of the area looks like an abrasion with a shallow crater. The nurse would document this ulcer as being at which stage?

Tremors are an expected side effect of rapidly acting bronchodilators. Tremors are a common side effect of short-acting β2-adrenergic (SABA) medications and not a reason to avoid using the SABA inhaler. Inhaled corticosteroids do not act rapidly to reduce dyspnea. Rapid inhalation is needed when using a DPI. The patient should hold the breath for 10 seconds after using inhalers.

Which information will the nurse include in the asthma teaching plan for a patient being discharged?

Antithyroid medications may take several months for full effect.

Which information will the nurse teach a 48-year-old patient who has been newly diagnosed with Graves' disease?

Offering the patient pain medication 30-45 minutes before ambulating

Which intervention by the nurse will help a postoperative patient with compliance in getting up to ambulate?

Reinforce the teaching about avoiding alcohol and caffeine for a client with chronic gastritis. Correct: Reinforcement of teaching done by the nurse is within the scope of practice for an LPN/LVN.

Which nursing action is best for the charge nurse to delegate to an experienced licensed practical nurse/vocational nurse (LPN/LVN)?

Collect a detailed diet history. The initial nursing action should be assessment of the patient's baseline dietary intake through a thorough diet history. The other actions may be appropriate, but assessment of the patient's baseline should occur first.

Which nursing action should the nurse take first in order to assist a patient with newly diagnosed stage 1 hypertension in making needed dietary changes?

ANS: D Muscle twitching and finger numbness indicate hypocalcemia, which may lead to tetany unless calcium gluconate is administered. Although the other findings should also be reported to the health care provider, they do not indicate complications that require rapid action.

20. Which assessment finding would the nurse need to report most quickly to the health care provider regarding a patient with acute pancreatitis? a. Nausea and vomiting b. Hypotonic bowel sounds c. Abdominal tenderness and guarding d. Muscle twitching and finger numbness

ANS: D Alcohol use is one of the most common risk factors for pancreatitis in the United States. Cigarette smoking, diabetes, and high-protein diets are not risk factors.

21. The nurse will ask a 64-year-old patient being admitted with acute pancreatitis specifically about a history of a. diabetes mellitus. b. high-protein diet. c. cigarette smoking. d. alcohol consumption.

ANS: C Pancreatic enzymes are used to help with digestion of nutrients and should be taken with every meal.

22. The nurse will teach a patient with chronic pancreatitis to take the prescribed pancrelipase (Viokase) a. at bedtime. b. in the morning. c. with each meal. d. for abdominal pain.

ANS: A The purpose of lactulose in the patient with cirrhosis is to lower ammonia levels and prevent encephalopathy. Although lactulose may be used to treat constipation, that is not the purpose for this patient. Lactulose will not decrease nausea and vomiting or lower bilirubin levels.

12. Which finding indicates to the nurse that lactulose (Cephulac) is effective for a 72-year-old man who has advanced cirrhosis? a. The patient is alert and oriented. b. The patient denies nausea or anorexia. c. The patient's bilirubin level decreases. d. The patient has at least one stool daily.

ANS: B After a laparoscopic cholecystectomy, the patient will have Band-Aids in place over the incisions. Patients are discharged the same (or next) day and have few restrictions on activities of daily living. Drainage from the incisions would be abnormal, and the patient should be instructed to call the health care provider if this occurs. A low-fat diet may be recommended for a few weeks after surgery but will not be a lifelong requirement.

23. The nurse recognizes that teaching a 44-year-old woman following a laparoscopic cholecystectomy has been effective when the patient states which of the following? a. "I can expect yellow-green drainage from the incision for a few days." b. "I can remove the bandages on my incisions tomorrow and take a shower." c. "I should plan to limit my activities and not return to work for 4 to 6 weeks." d. "I will need to maintain a low-fat diet for life because I no longer have a gallbladder.

ANS: B After a laparoscopic cholecystectomy, the patient will have Band-Aids in place over the incisions. Patients are discharged the same (or next) day and have few restrictions on activities of daily living.

23. The nurse recognizes that teaching a 44-year-old woman following a laparoscopic cholecystectomy has been effective when the patient states which of the following? a. "I can expect yellow-green drainage from the incision for a few days." b. "I can remove the bandages on my incisions tomorrow and take a shower." c. "I should plan to limit my activities and not return to work for 4 to 6 weeks." d. "I will need to maintain a low-fat diet for life because I no longer have a gallbladder."

ANS: B Asterixis indicates that the patient has hepatic encephalopathy, and hepatic coma may occur. The ascites and weight gain indicate the need for treatment but not as urgently as the changes in neurologic status.

24. The nurse is caring for a 73-year-old man who has cirrhosis. Which data obtained by the nurse during the assessment will be of most concern? b. The patient's hands flap back and forth when the arms are extended. c. The patient has ascites and a 2-kg weight gain from the previous day.

ANS: C The patient with acute pancreatitis is at risk for hypocalcemia, and the assessment data indicate a positive Trousseau's sign. The health care provider should be notified after the nurse checks the patient's calcium level.

27. When taking the blood pressure (BP) on the right arm of a patient with severe acute pancreatitis, the nurse notices carpal spasms of the patient's right hand. Which action should the nurse take next? c. Check the calcium level in the chart. d. Notify the health care provider immediately.

ANS: D A palpable abdominal mass may indicate the presence of a pancreatic abscess, which will require rapid surgical drainage to prevent sepsis. Absent bowel sounds, abdominal tenderness, and left upper quadrant pain are common in acute pancreatitis

29. Which assessment finding is of most concern for a 46-year-old woman with acute pancreatitis? a. Absent bowel sounds b. Abdominal tenderness c. Left upper quadrant pain d. Palpable abdominal mass

ANS: B Providing oral hygiene is within the scope of UAP. Assessments and assisting patients to choose therapeutic diets are nursing actions that require higher-level nursing education and scope of practice and would be delegated to licensed practical/vocational nurses (LPNs/LVNs) or RNs.

39. For a patient with cirrhosis, which of the following nursing actions can the registered nurse (RN) delegate to unlicensed assistive personnel (UAP)? b. Providing oral hygiene after a meal d. Assisting the patient to choose the diet

ANS: D The patient with chronic pancreatitis does not present an infection risk to the immunosuppressed patient who had a liver transplant. The other patients either are at risk for infection or currently have an infection, which will place the immunosuppressed patient at risk for infection.

44. A nurse is considering which patient to admit to the same room as a patient who had a liver transplant 3 weeks ago and is now hospitalized with acute rejection. best choice? a. Patient who is receiving chemotherapy for liver cancer d. Patient who requires pain management for chronic pancreatitis

ANS: C The area around the umbilicus should be indicated. Cullen's sign consists of ecchymosis around the umbilicus. Cullen's sign occurs because of seepage of bloody exudates from the inflamed pancreas and indicates severe acute pancreatitis.

45. In reviewing the medical record shown in the accompanying figure for a patient admitted with acute pancreatitis, the nurse sees that the patient has a positive Cullen's sign. Indicate the area where the nurse will assess for this change. a. 1 b. 2 c. 3 Bruising around the umbilicus d. 4

ANS: A, B, D The recommendations for hepatitis B exposure include both vaccination and immune globulin administration. In addition, baseline testing for hepatitis B antibodies will be needed. Interferon and oral antivirals are not used for hepatitis B prophylaxis.

46. After an unimmunized individual is exposed to hepatitis B through a needle-stick injury, which actions will the nurse plan to take (select all that apply)? a. ADMINISTER HEP B VACCINE b. TEST FOR ANTIBODIES FOR HEP B c. Teach about α-interferon therapy. d. GIVE HEP B IMMUNOGLOBIN

D All options are important nursing actions in the care of a trauma client. However, airway always comes first. The client must have a patent airway, or other interventions will not be helpful.

A client is brought to the emergency department after being shot in the abdomen and is hemorrhaging heavily. Which action by the nurse is the priority? a. Draw blood for type and crossmatch. b. Start two large IVs for fluid resuscitation. c. Obtain vital signs and assess skin perfusion. d. Assess and maintain a patent airway.

anti-hepatitis A virus immunoglobulin M (anti-HAV IgM). Hepatitis A is transmitted through the oral-fecal route, and antibody to HAV IgM appears during the acute phase of hepatitis A. The patient would not have antigen for hepatitis B or antibody for hepatitis D. Anti-HAV IgG would indicate past infection and lifelong immunity.

A 24-year-old female contracts hepatitis from contaminated food. During the acute (icteric) phase of the patient's illness, the nurse would expect serologic testing to reveal

3. It is most likely that the client is experiencing an adverse effect of the antacid. Antacids with aluminum salt products, such as aluminum hydroxide, form insoluble salts in the body. These precipitate and accumulate in the intestines, causing constipation. Increasing dietary fiber intake or daily exercise may be a beneficial lifestyle change for the client but is not likely to relieve the constipation caused by the aluminum hydroxide. Constipation, in isolation from other symptoms, is not a sign of a bowel obstruction.

A client has been taking aluminum hydroxide (Amphojel) 30 mL six times per day at home to treat his peptic ulcer. He tells the nurse that he has been unable to have a bowel movement for 3 days. Based on this information, the nurse would determine that which of the following is the most likely cause of the client's constipation? 1. The client has not been including enough fiber in his diet. 2. The client needs to increase his daily exercise. 3. The client is experiencing an adverse effect of the aluminum hydroxide. 4. The client has developed a gastrointestinal obstruction.

B Clients with irritable bowel syndrome are advised to eat a high-fiber diet (30 to 40 grams a day), with 8 to 10 cups of liquid daily. This selection has the highest fiber content. They should avoid alcohol, caffeine, and other gastric irritants.

A client has irritable bowel syndrome. Which menu selections by this client indicate good understanding of dietary teaching? a. Tuna salad on white bread, cup of applesauce, glass of diet cola b. Broiled chicken with brown rice, steamed green beans, glass of apple juice c. Grilled cheese sandwich, small ripe banana, cup of hot tea with lemon d. Grilled steak, green beans, dinner roll with butter, cup of coffee with cream

ANS: B Serum and urine amylase levels are elevated when the pancreas becomes inflamed. One cause of pancreatitis is gallbladder disease; another causative factor is alcohol intake. The nurse should tactfully explore this subject with the client before documenting the findings and notifying the provider. The client may need to be NPO or on clear liquids, but the nurse does not have enough information yet to determine this.

A client is admitted for suspected cholecystitis. On reviewing laboratory results, the nurse notes that the client's amylase is elevated. Which action by the nurse is best? a. Document the finding in the chart. b. Ask the client about drinking habits. c. Notify the health care provider. d. Place the client on clear liquids.

4. The body reacts to perforation of an ulcer by immobilizing the area as much as possible. This results in boardlike abdominal rigidity, usually with extreme pain. Perforation is a medical emergency requiring immediate surgical intervention because peritonitis develops quickly after perforation. An intestinal obstruction would not cause midepigastric pain. The development of additional ulcers or esophageal inflammation would not cause a rigid, boardlike abdomen.

A client is admitted to the hospital after vomiting bright red blood and is diagnosed with a bleeding duodenal ulcer. The client develops a sudden, sharp pain in the midepigastric region along with a rigid, boardlike abdomen. These clinical manifestations most likely indicate which of the following? 1. An intestinal obstruction has developed. 2. Additional ulcers have developed. 3. The esophagus has become inflamed. 4. The ulcer has perforated.

Immediately place the client on Airborne Precautions. This "allergy test" is actually a positive tuberculosis test. The client should be placed on Airborne Precautions immediately. The other options do not take priority over preventing the spread of the disease.

A client is admitted with suspected pneumonia from the emergency department. The client went to the primary care provider a "few days ago" and shows the nurse the results of what the client calls "an allergy test," as shown below: What action by the nurse takes priority?

C Staphylococcus can be found in meat and dairy products and can be transmitted to people. Food poisoning occurs, especially if foods are left unrefrigerated over a period of time. Symptoms of Staphylococcus food poisoning include sudden onset of vomiting, abdominal cramping, and diarrhea within 2 to 4 hours. The client's symptoms are not consistent with infection by the other microorganisms.

A client is brought to the emergency department with an abrupt onset of vomiting, abdominal cramping, and diarrhea 2 hours after eating food at a picnic. Which infectious microorganism does the nurse suspect as the probable cause? a. Salmonella b. Giardia lamblia c. Staphylococcus aureus d. Clostridium botulinum

ANS: A Spasm of the hand when a blood pressure cuff is applied (Trousseau's sign) is indicative of hypocalcemia. The client's calcium level is low. The sodium level is high, but that is not related to Trousseau's sign. Creatinine and potassium levels are normal.

A client is hospitalized with acute pancreatitis. The nursing assistant reports to the nurse that when a blood pressure cuff was applied, the client's hand had a spasm. Which additional finding does the nurse correlate with this condition? a. Serum calcium, 5.8 mg/dL b. Serum sodium, 166 mEq/L c. Serum creatinine, 0.9 mg/dL d. Serum potassium, 4.2 mEq/dL

4. Antacids are most effective if taken 1 to 3 hours after meals and at bedtime. When an antacid is taken on an empty stomach, the duration of the drug's action is greatly decreased. Taking antacids 1 to 3 hours after a meal lengthens the duration of action, thus increasing the therapeutic action of the drug. Antacids should be administered about 2 hours after other medications to decrease the chance of drug interactions. It is not necessary to decrease fluid intake when taking antacids. If antacids are taken more frequently than recommended, the likelihood of developing adverse effects increases. Therefore, the client should not take antacids as often as desired to control pain.

A client is taking an antacid for treatment of a peptic ulcer. Which of the following statements best indicates that the client understands how to correctly take the antacid? 1. "I should take my antacid before I take my other medications." 2. "I need to decrease my intake of fluids so that I don't dilute the effects of my antacid." 3. "My antacid will be most effective if I take it whenever I experience stomach pains." 4. "It is best for me to take my antacid 1 to 3 hours after meals."

B The first bowel movement after hemorrhoidectomy can be painful enough to induce syncope. The nurse should stay with the client. The nurse should instruct clients who are discharged the same day to have someone nearby when they have their first postoperative bowel movement. Making sure needed items are within reach is an important nursing action too, but it does not take priority over client safety. The other two actions are not needed in this situation.

A client post-hemorrhoidectomy feels the need to have a bowel movement. Which action by the nurse is best? a. Have the client use the bedside commode. b. Stay with the client, providing privacy. c. Make sure toilet paper and the call light are in reach. d. Plan to send a stool sample to the laboratory.

A The nurse should collaborate with the ostomy nurse to help the client and her husband work through intimacy issues. The nurse should not minimize the client's concern about her husband with ways to hide the ostomy. The client will not hurt the stoma by becoming intimate with her husband.

A client tells the nurse that her husband is repulsed by her colostomy and refuses to be intimate with her after surgery. Which is the nurse's best response? a. "Let's talk to the ostomy nurse to help you and your husband work through this." b. "You could try to wear longer lingerie that will better hide the ostomy appliance." c. "You should empty the pouch first so it will be less noticeable for your husband." d. "If you are not careful, you can hurt the stoma if you engage in sexual activity."

A The stool from an ascending colostomy can be expected to remain liquid because little large bowel is available to reabsorb the liquid from the stool. The provider may be notified, but this is not the best response from the nurse. Liquid stool from an ascending colostomy will not become firmer with the addition of fiber to the client's diet or with the passage of time.

A client who has had a colostomy placed in the ascending colon expresses concern that the effluent collected in the colostomy pouch has remained liquid for 2 weeks after surgery. Which is the nurse's best response? a. "This is normal for your type of colostomy." b. "I will let the health care provider know, so that it can be assessed." c. "You should add extra fiber to your diet to stop the diarrhea." d. "Your stool will become firmer over the next few weeks."

C A negative result does not completely rule out the possibility of colon cancer. To determine whether the client has colon cancer, a colonoscopy should be performed, so the entire colon can be visualized and a tissue sample taken for biopsy. The client need not wait a week before the colonoscopy. Two negative fecal occult blood tests do not rule out the presence of colorectal cancer (CRC).

A client who has had fecal occult blood testing tells the nurse that the test was negative for colon cancer and wishes to cancel a colonoscopy scheduled for the next day. Which is the nurse's best response? a. "I will call and cancel the test for tomorrow." b. "You need two negative fecal occult blood tests." c. "This does not rule out the possibility of colon cancer." d. "You should wait at least a week to have the colonoscopy."

A Fistulas place the client with Crohn's disease at risk for hypokalemia, which can lead to serious dysrhythmias. This potassium level is low and should cause the nurse to intervene. The white blood cell count is normal. The other two findings are abnormal and also warrant intervention, but the potassium takes priority.

A client with Crohn's disease has a draining fistula. Which finding leads the nurse to intervene most rapidly? a. Serum potassium of 2.6 mEq/L b. The client not wanting to eat anything c. White blood cell count of 8200/mm3 d. The client losing 3 pounds in a week

D A change in the nature and timing of abdominal pain in a client with a bowel obstruction can signal peritonitis or perforation. The nurse should immediately check for rebound tenderness and the absence of bowel sounds. The nurse need not measure abdominal girth. The nurse may help the client to the knee-chest position for comfort, but this is not the priority action. The nurse should not medicate the client until the physician has been notified of the change in his or her condition.

A client with a mechanical bowel obstruction reports that abdominal pain, which was previously intermittent and colicky, is now more constant. Which is the priority action of the nurse? a. Measure the abdominal girth. b. Place the client in a knee-chest position. c. Medicate the client with an opioid analgesic. d. Assess for bowel sounds and rebound tenderness.

4. It would be most effective for the client to develop a health maintenance plan that incorporates regular periods of physical and mental rest in the daily schedule. Strategies should be identified to deal with the types of physical and mental stressors that the client needs to cope with in the home and work environments. Scheduling physical activity to occur only in the morning would not be restful or practical. There is no need for the client to avoid yard work or jogging if these activities are not stressful.

A client with a peptic ulcer has been instructed to avoid intense physical activity and stress. Which strategy should the client incorporate into the home care plan? 1. Conduct physical activity in the morning so that he can rest in the afternoon. 2. Have the family agree to perform the necessary yard work at home. 3. Give up jogging and substitute a less demanding hobby. 4. Incorporate periods of physical and mental rest in his daily schedule.

1, 2, 3, 4. The nurse should encourage the client to reduce stimulation that may enhance gastric secretion. The nurse can also advise the client to utilize health practices that will prevent recurrences of ulcer pain, such as avoiding fatigue and elimination of smoking. Eating small, frequent meals helps to prevent gastric distention if not actively bleeding and decreases distension and release of gastrin. Medications should be administered promptly to maintain optimum levels. After awakening during the night, the client should eat a small snack and return to bed, keeping the head of the bed elevated for an hour after eating. It is not necessary to stay away from crowded areas.

A client with a peptic ulcer reports epigastric pain that frequently awakens her during the night. The nurse should instruct the client to do which activities? Select all that apply. 1. Obtain adequate rest to reduce stimulation. 2. Eat small, frequent meals throughout the day. 3. Take all medications on time as ordered. 4. Sit up for one hour when awakened at night. 5. Stay away from crowded areas.

4. Histamine-2 (H2) receptor antagonists, such as ranitidine, reduce gastric acid secretion. Antisecretory, or proton-pump inhibitors, such as omeprazole (Prilosec), help ulcers heal quickly in 4 to 8 weeks. Cytoprotective drugs, such as sucralfate (Carafate), protect the ulcer surface against acid, bile, and pepsin. Antacids reduce acid concentration and help reduce symptoms.

A client with peptic ulcer disease is taking ranitidine (Zantac). What is the expected outcome of this drug? 1. Heal the ulcer. 2. Protect the ulcer surface from acids. 3. Reduce acid concentration. 4. Limit gastric acid secretion.

2, 3. The symptoms of nausea and dizziness in a client with peptic ulcer disease may be indicative of hemorrhage and should not be ignored. The appropriate nursing actions at this time are for the nurse to monitor the client's vital signs and notify the physician of the client's symptoms. To administer an antacid hourly or to wait 1 hour to reassess the client would be inappropriate; prompt intervention is essential in a client who is potentially experiencing a gastrointestinal hemorrhage. The nurse would notify the physician of assessment findings and then initiate oxygen therapy if ordered by the physician.

A client with peptic ulcer disease reports that he has been nauseated most of the day and is now feeling light-headed and dizzy. Based upon these findings, which nursing actions would be most appropriate for the nurse to take? Select all that apply. 1. Administering an antacid hourly until nausea subsides. 2. Monitoring the client's vital signs. 3. Notifying the physician of the client's symptoms. 4. Initiating oxygen therapy. 5. Reassessing the client in an hour.

2. Black, tarry stools are an important warning sign of bleeding in peptic ulcer disease. Digested blood in the stool causes it to be black. The odor of the stool is very offensive. Clients with peptic ulcer disease should be instructed to report the incidence of black stools promptly to their primary health care provider. The data do not support the other diagnoses.

A client with peptic ulcer disease tells the nurse that he has black stools, which he has not reported to his physician. Based on this information, which nursing diagnosis would be appropriate for this client? 1. Ineffective coping related to fear of diagnosis of chronic illness. 2. Deficient knowledge related to unfamiliarity with significant signs and symptoms. 3. Constipation related to decreased gastric motility. 4. Imbalanced nutrition: Less than body requirements related to gastric bleeding.

Take a shower immediately, lathering and rinsing several times Have them to cleanse the area to remove the sap that may cause irritation. Calamine lotion may be used if dermatitis develops.

A clients contact the ED of a healthcare facility and tells the nurse that he has come in contact with poison ivy while cleaning his backyard. The client ells the nurse that he can not see anything on the skin and asks the nurse what to do. What is the best response by the nurse?

Assess the patient for anxiety. Anxiety can interfere with learning, coping, and cooperation. The nurse should assess the patient for anxiety. The other actions are appropriate too, and can be included in the teaching plan, but effective teaching cannot occur if the patient is highly anxious.

A clinic nurse is teaching a patient prior to surgery. The patient does not seem to comprehend the teaching, forgets a lot of what is said, and asks the same questions again and again. What action by the nurse is best?

B Lubiprostone (Amitiza) is approved only for use in women. The other statements are not accurate.

A middle-aged male client has irritable bowel syndrome that has not responded well to diet changes and bulk-forming laxatives. He asks the nurse about the new drug lubiprostone (Amitiza). What information does the nurse provide him? a. "This drug is investigational right now for irritable bowel syndrome." b. "Unfortunately, this drug is approved only for use in women." c. "Lubiprostone works well only in a small fraction of irritable bowel cases." d. "Let's talk to your health care provider about getting you a trial prescription."

ANS: B Acetaminophen (Tylenol) can cause liver damage if taken in large amounts. Clients should be taught not to exceed 4000 mg/day of acetaminophen. The nurse should teach the client about this limitation and should explore other drug options with the client to manage his or her arthritis pain. Two glasses of wine each week, a cousin with liver cancer, and the hepatitis vaccine do not place the client at risk for a liver disorder, and therefore do not require any health teaching.

A nose obtains a clients's health history at a community health clinic. Which statement alerts the nurse to prove health teaching to this client? a. "I drink two glasses of red wine each week." b. "I take a lot of Tylenol for my arthritis pain." c. "I have a cousin who died of liver cancer." d. "I got a hepatitis vaccine before traveling."

ANS: B, E, F Elevated INR and PT are indications of clotting disturbances and alert the nurse to the increased possibility of hemorrhage. Elevated ammonia levels increase the client's confusion. The other values are abnormal and associated with liver disease but do not necessarily place the client at increased risk for complications.

A nurse assesses a client who has liver disease. Which laboratory findings should the nurse recognize as potentially causing complications of this disorder? (Select all that apply.) a. Elevated aspartate transaminase b. Elevated international normalized ratio (INR) c. Decreased serum globulin levels d. Decreased serum alkaline phosphatase e. Elevated serum ammonia f. Elevated prothrombin time (PT)

ANS: C - Although the client with severe diarrhea may experience skin irritation and hypovolemia, the client is most at risk for cardiac dysrhythmias secondary to potassium and magnesium loss from severe diarrhea. The client should have her or his electrolyte levels monitored, and electrolyte replacement may be necessary. Oral mucosa inspection, recent dietary intake, and abdominal percussion are important parts of physical assessment but are lower priority for this client than heart rate and rhythm.

A nurse assesses a client who has ulcerative colitis and severe diarrhea. Which assessment should the nurse complete first? a. Inspection of oral mucosa b. Recent dietary intake c. Heart rate and rhythm d. Percussion of abdomen

ANS: B - A client with botulism is at risk for respiratory failure. This clients respiratory rate is slow, which could indicate impending respiratory distress or failure. The nurse should remain with the client while another nurse notifies the provider. The nurse should monitor and document the IV infusion per protocol, but this client does not require additional intravenous fluids. Allowing the client to rest or checking the clients blood glucose and administering orange juice are not appropriate actions.

A nurse assesses a client who is hospitalized for botulism. The clients vital signs are temperature: 99.8 F (37.6 C), heart rate: 100 beats/min, respiratory rate: 10 breaths/min, and blood pressure: 100/62 mm Hg. Which action should the nurse take? a. Decrease stimulation and allow the client to rest. b. Stay with the client while another nurse calls the provider. c. Increase the clients intravenous fluid replacement rate. d. Check the clients blood glucose and administer orange juice.

ANS: D Mid-sternal chest pain is indicative of acute angina or myocardial infarction, which can be precipitated by vasopressin. Nausea and vomiting, headache, and vertigo and syncope are not side effects of vasopressin.

A nurse assesses a client who is prescribed an infusion of vasopressin (Pitressin) for bleeding esophageal varices. Which clinical manifestation should alert the nurse to a serious adverse effect? a. Nausea and vomiting b. Frontal headache c. Vertigo and syncope d. Mid-sternal chest pain

ANS: A Complications of a Whipple procedure include secretions that drain from a fistula and peritonitis. Absent bowel sounds, pain at the incision site, and NG tube drainage are normal postoperative findings.

A nurse assesses a client who is recovering from a Whipple procedure. Which assessment finding alerts the nurse to urgently contact the health care provider? a. Drainage from a fistula b. Absent bowel sounds c. Pain at the incision site d. Nasogastric (NG) tube drainage

ANS: B, C, D, E Myocardial infarction (chest pain), pulmonary embolism (shortness of breath), adynamic ileus (lack of bowel sounds or flatus), and renal failure (urine output of 20 mL/6 hr) are just some of the complications for which the nurse must assess the client after the Whipple procedure. Clay-colored stools are associated with cholecystitis and are not a complication of a Whipple procedure.

A nurse assesses a client who is recovering from a Whipple procedure. Which clinical manifestations alert the nurse to a complication from this procedure? (Select all that apply.) a. Clay-colored stools b. Substernal chest pain c. Shortness of breath d. Lack of bowel sounds or flatus e. Urine output of 20 mL/6 hr

ANS: A Rapid removal of ascetic fluid causes decreased abdominal pressure, which can contribute to hypovolemia. This can be manifested by a decrease in urine output to below 30 mL/hr. A slight increase in systolic blood pressure is insignificant. A decrease in respiratory rate indicates that breathing has been made easier by the procedure. The nurse would expect the client's weight to drop as fluid is removed. Six kilograms is less than 3 pounds and is expected.

A nurse assesses a client who is recovering from a paracentesis 1 hour ago. Which assessment finding requires action by the nurse? a. Urine output via indwelling urinary catheter is 20 mL/hr b. Blood pressure increases from 110/58 to 120/62 mm Hg c. Respiratory rate decreases from 18 to 14 breaths/min d. A decrease in the client's weight by 6 kg

ANS: A, B, E When assessing a client with suspected cirrhosis, the nurse should ask about alcohol consumption, including amount and frequency; sexual history and orientation (specifically men having sex with men); illicit drug use; history of tattoos; and history of military service, incarceration, or work as a firefighter, police officer, or health care provider. A family history of cancer and work as a plumber do not put the client at risk for cirrhosis.

A nurse assesses a male client who has symptoms of cirrhosis. Which questions should the nurse ask to identify potential factors contributing to this laboratory result? (Select all that apply.) a. "How frequently do you drink alcohol?" b. "Have you ever had sex with a man?" c. "Do you have a family history of cancer?" d. "Have you ever worked as a plumber?" e. "Were you previously incarcerated?"

Bradycardia and loss of eyebrow hair Lithium causes decreased synthesis of thyroid hormone, so the nurse assesses for signs of hypothyroidism including bradycardia and loss of eyebrow hair. Heat intolerance and weight loss indicate hyperthyroidism. Chvostek's and Trousseau's signs are indicative of hypocalcemia and hypoparathyroidism. Fractures and decreased bone density can indicate hyperparathyroidism.

A nurse assesses a patient who takes lithium. Which assessment finding should alert the nurse to a side effect of this therapy?

Assess the patient's medications. Pacemaker cells in the conduction system decrease in number as a person ages, resulting in bradycardia. The nurse would check the medication reconciliation for medications that might cause such a drop in heart rate, and then would inform the healthcare provider. Documentation is important, but it is not the priority action. The heart rate is not low enough for atropine or an external pacemaker to be needed.

A nurse assesses an older adult patient who has multiple chronic diseases. The patient's heart rate is 48 beats/min. What action would the nurse take first?

A 20-year-old college student who has had several sexual partners Hepatitis B can be spread through sexual contact, needle sharing, needle sticks, blood transfusions, hemodialysis, acupuncture, and the maternal-fetal route. A person with multiple sexual partners has more opportunities to contract the infection. Hepatitis B is not transmitted through medications, casual contact with other travelers, or raw shellfish. Although an overdose of acetaminophen can cause liver cirrhosis, this is not associated with hepatitis B. Hepatitis E is found most frequently in international travelers. Hepatitis A is spread through ingestion of contaminated shellfish.

A nurse assesses clients at a community health fair. Which client is at greatest risk for the development of hepatitis B?

ANS: C Clients who have chronic cirrhosis may have alienated relatives over the years because of substance abuse. The nurse should assist the client to identify a friend, neighbor, or person in his or her recovery group for support. The nurse should not minimize the client's concerns by brushing off the client's comment. Attending AA may be appropriate, but this response doesn't address the client's concern. Making peace with the client's family may not be possible. This statement is not client-centered.

A nurse cares for a client who has chronic cirrhosis from substance abuse. The client states, "All of my family hates me." How should the nurse respond? a. "You should make peace with your family." b. "This is not unusual. My family hates me too." c. "I will help you identify a support system." d. "You must attend Alcoholics Anonymous."

ANS: D Maintaining airway patency is the primary nursing intervention for this client. The nurse suctions oral secretions to prevent aspiration and occlusion of the airway. The client usually is intubated and mechanically ventilated during this treatment. The client should be sedated, balloon pressure should be maintained between 15 and 20 mm Hg, and the lumen can be irrigated with saline or tap water. However, these are not a higher priority than airway patency.

A nurse cares for a client who is hemorrhaging from bleeding esophageal varices and has an esophagogastric tube. Which action should the nurse take first? a. Sedate the client to prevent tube dislodgement. b. Maintain balloon pressure at 15 and 20 mm Hg. c. Irrigate the gastric lumen with normal saline. d. Assess the client for airway patency.

ANS: A The purpose of administering lactulose to this client is to help ammonia leave the circulatory system through the colon. Lactulose draws water into the bowel with its high osmotic gradient, thereby producing a laxative effect and subsequently evacuating ammonia from the bowel. The client must understand that this is an expected and therapeutic effect for him or her to remain compliant. The nurse should not suggest administering anything that would decrease the excretion of ammonia or holding the medication. There is no need to send a stool specimen to the laboratory because diarrhea is the therapeutic response to this medication.

A nurse cares for a client who is prescribed lactulose (Heptalac). The client states, "I do not want to take this medication because it causes diarrhea." How should the nurse respond? a. "Diarrhea is expected; that's how your body gets rid of ammonia." b. "You may take Kaopectate liquid daily for loose stools." c. "Do not take any more of the medication until your stools firm up." d. "We will need to send a stool specimen to the laboratory."

Postoperative care for a client recovering from an open Whipple procedure should include placing the client in a semi-Fowler's position to reduce tension on the suture line and anastomosis sites, setting the nasogastric tube to low suction to remove free air buildup and pressure, assessing vital signs frequently to assess fluid and electrolyte complications, and providing intravenous fluids.

A nurse cares for a client who is recovering from an open Whipple procedure. Which action should the nurse take? a. Clamp the nasogastric tube. b. Place the client in semi-Fowler's position. c. Assess vital signs once every shift. d. Provide oral rehydration.

ANS: C Paralytic ileus is a common complication of acute pancreatitis. The client should not eat until this has resolved. Bowel sounds and decreased pain are not reliable indicators of peristalsis. Instead, the nurse should assess for passage of flatus or bowel movement.

A nurse cares for a client with acute pancreatitis. The client states, "I am hungry." How should the nurse reply? a. "Is your stomach rumbling or do you have bowel sounds?" b. "I need to check your gag reflex before you can eat." c. "Have you passed any flatus or moved your bowels?" d. "You will not be able to eat until the pain subsides."

ANS: B A low-protein diet is ordered when serum ammonia levels increase and/or the client shows signs of PSE. A low-protein diet helps reduce excessive breakdown of protein into ammonia by intestinal bacteria. Encephalopathy is caused by excess ammonia. A low-protein diet has no impact on restoring liver function. Increasing the client's dietary protein will cause complications of liver failure and should not be suggested. Increased intravascular protein will help prevent ascites, but clients with liver failure are not able to effectively synthesize dietary protein.

A nurse cares for a client with hepatic portal-systemic encephalopathy (PSE). The client is thin and cachectic in appearance, and the family expresses distress that the client is receiving little dietary protein. How should the nurse respond? a. "A low-protein diet will help the liver rest and will restore liver function." b. "Less protein in the diet will help prevent confusion associated with liver failure." c. "Increasing dietary protein will help the client gain weight and muscle mass." d. "Low dietary protein is needed to prevent fluid from leaking into the abdomen."

ANS: B Although family members may be afraid that they will contract hepatitis C, the nurse should educate the client's family about how the virus is spread. Viral hepatitis, or hepatitis C, is spread via blood-to-blood transmission and is associated with illicit IV drug needle sharing, blood and organ transplantation, accidental needle sticks, unsanitary tattoo equipment, and sharing of intranasal cocaine paraphernalia. Wearing a gown and gloves will not decrease the transmission of this virus. Hepatitis C is not spread through casual contact or a fecal specimen. The nurse would be violating privacy laws by sharing the client's status with the brother.

A nurse cares for a client with hepatitis C. The client's brother states, "I do not want to contract this infection, so I will not go into his hospital room." How should the nurse respond? a. "If you wear a gown and gloves, you will not get this virus." b. "Viral hepatitis is not spread through casual contact." c. "This virus is only transmitted through a fecal specimen." d. "I can give you an update on your brother's status from here."

ANS: A The client with hepatopulmonary syndrome is often dyspneic. Because the oxygen saturation is not significantly low, the nurse should first allow the client to sit upright to see if that helps. If the client remains dyspneic, or if the oxygen saturation drops further, the nurse should investigate adding humidity to the oxygen and seeing whether the client will tolerate that. The other two options may be beneficial, but they are not the best choices. If the client is comfortable, his or her agitation will decrease; this will improve respiratory status.

A nurse cares for a client with hepatopulmonary syndrome who is experiencing dyspnea with oxygen saturations at 92%. The client states, "I do not want to wear the oxygen because it causes my nose to bleed. Get out of my room and leave me alone!" Which action should the nurse take? a. Instruct the client to sit in as upright a position as possible. b. Add humidity to the oxygen and encourage the client to wear it. c. Document the client's refusal, and call the health care provider. d. Contact the provider to request an extra dose of the client's diuretic.

ANS: B, C, D The client should be placed in a private room and dirty linens kept in the client's room until the radiation source is removed. The nurse should wear a lead apron while providing care, ensuring that the apron always faces the client. The nurse should also bundle care to minimize exposure to the client. Transmission-Based Precautions will not protect the nurse from the implanted radioactive iodine seeds.

A nurse cares for a client with pancreatic cancer who is prescribed implanted radioactive iodine seeds. Which actions should the nurse take when caring for this client? (Select all that apply.) a. Dispose of dirty linen in a red "biohazard" bag. b. Place the client in a private room. c. Wear a lead apron when providing client care. d. Bundle care to minimize exposure to the client. e. Initiate Transmission-Based Precautions.

ANS: B, C, D During the healing phase of pancreatitis, the client should be provided small, frequent, moderate- to high-carbohydrate, high-protein, low-fat meals. Protein shakes can be provided to supplement the diet. Foods and beverages should not contain caffeine and should be bland.

A nurse collaborates with an unlicensed assistive personnel (UAP) to provide care for a client who is in the healing phase of acute pancreatitis. Which statements focused on nutritional requirements should the nurse include when delegating care for this client? (Select all that apply.) a. "Do not allow the client to eat between meals." b. "Make sure the client receives a protein shake." c. "Do not allow caffeine-containing beverages." d. "Make sure the foods are bland with little spice." e. "Do not allow high-carbohydrate food items."

ANS: A, C, D Clients with advanced cirrhosis often have pruritus. Lotion will help decrease itchiness from dry skin. A soft toothbrush should be used to prevent gum bleeding, and the client's nails should be trimmed short to prevent the client from scratching himself or herself. These clients should use cool, not warm, water on their skin, and should not use excessive amounts of soap.

A nurse delegates hygiene care for a client who has advanced cirrhosis to an unlicensed nursing personnel (UAP). Which statements should the nurse include when delegating this task to the UAP? (Select all that apply.) a. "Apply lotion to the client's dry skin areas." b. "Use a basin with warm water to bathe the client." c. "For the client's oral care, use a soft toothbrush." d. "Provide clippers so the client can trim the fingernails." e. "Bathe with antibacterial and water-based soaps."

Assess the right leg for pulses, skin color, and temperature. A patient with an ulcer on the foot would be assessed for interruption in arterial flow to the area. This begins with the assessment of pulses and color and temperature of the skin. The nurse can also assess for pulses noninvasively with a Doppler flowmeter if unable to palpate with his or her fingers. Tests to determine nutritional status and risk assessment would be completed after the initial assessment is done. Wound cultures are done after it has been determined that drainage, odor, and other risks for infection are present. Elevation of the foot would impair the ability of arterial blood to flow to the area.

A nurse is caring for a patient who has a pressure ulcer on the right ankle. Which action would the nurse take first?

Assess the client's level of consciousness. Assessing the client's level of consciousness will be most important because it will show how the client is responding to the presence of the infection. Although it will be important for the nurse to encourage the client to turn, cough, and frequently breathe deeply; raise the head of the bed; increase oral fluid intake; and humidify the oxygen administered, none of these actions will be as important as assessing the level of consciousness. Also, the client who has a pulmonary infection may not be able to cough effectively if an area of abscess is present.

A nurse is caring for an older adult client who has a pulmonary infection. Which action should the nurse take first?

A, C, D, E Rotavirus is more common among the youngest of clients, not drinking water while swimming can help prevent E. coli infection, people with botulism need to be hospitalized to monitor for respiratory failure and paralysis, and parasitic diseases may take up to 2 weeks to become symptomatic. The other statements are not accurate.

A nurse is teaching a community group about food poisoning and gastroenteritis. Which statements by the nurse are accurate? (Select all that apply.) a. Rotavirus is more common among infants and younger children. b. Escherichia coli diarrhea is transmitted by contact with infected animals. c. Don't drink water when swimming to prevent E. coli infection. d. All clients with botulism require hospitalization. e. Parasitic diseases may not show up for 1 to 2 weeks after infection.

ANS: A, C, E Clients who are prescribed NPO while experiencing an acute pancreatitis episode may need enteral or parenteral nutrition. The nurse should collaborate with the registered dietitian, clinical pharmacist, and health care provider to plan and implement the more appropriate nutritional interventions. The nursing assistant and certified herbalist would not assist with this clinical decision.

A nurse plans care for a client who has acute pancreatitis and is prescribed nothing by mouth (NPO). With which health care team members should the nurse collaborate to provide appropriate nutrition to this client? (Select all that apply.) a. Registered dietitian b. Nursing assistant c. Clinical pharmacist d. Certified herbalist e. Health care provider

ANS: A, C, D Care for a client who has hepatopulmonary syndrome should include oxygen therapy, the head of bed elevated at least 30 degrees or as high as the client wants to improve breathing, elevated feet to decrease dependent edema, and daily weights. There is no need to place the client in a prone position, on the client's stomach. Although physical therapy may be helpful to a client who has been hospitalized for several days, physical therapy is not an intervention specifically for hepatopulmonary syndrome.

A nurse plans care for a client who has hepatopulmonary syndrome. Which interventions should the nurse include in this client's plan of care? (Select all that apply.) a. Oxygen therapy b. Prone position c. Feet elevated on pillows d. Daily weights e. Physical therapy

ANS: A A client recovering from chronic pancreatitis should be limited to one floor until strength and activity increase. The client will need a bathroom on the same floor for frequent defecation. Assessing pulse rate and preparation of meals is not specific to chronic pancreatitis. Although the client should be encouraged to stop drinking alcoholic beverages, asking about alcohol availability is not adequate to assess this client's safety.

A nurse prepares to discharge a client with chronic pancreatitis. Which question should the nurse ask to ensure safety upon discharge? a. "Do you have a one- or two-story home?" b. "Can you check your own pulse rate?" c. "Do you have any alcohol in your home?" d. "Can you prepare your own meals?"

ANS: A - The client should drink plenty of fluids to prevent dehydration. Milk products may not be tolerated. Caffeinated beverages increase intestinal motility and should be avoided.

A nurse teaches a client who has viral gastroenteritis. Which dietary instruction should the nurse include in this clients teaching? a. Drink plenty of fluids to prevent dehydration. b. You should only drink 1 liter of fluids daily. c. Increase your protein intake by drinking more milk. d. Sips of cola or tea may help to relieve your nausea.

ANS: B, D, F The client should be advised to stay sober, and AA is a great resource. The client requires a low-fat diet, and cooking spray is low in fat compared with butter or margarine. If the client smokes, he or she must stop because nicotine can precipitate an exacerbation. A nicotine patch may help the client quit smoking. The client must rest until his or her strength returns. The client requires high carbohydrates and calories for healing; complex carbohydrates are not preferred over simple ones. Dairy products do not cause a problem.

A nurse teaches a client who is recovering from acute pancreatitis. Which statements should the nurse include in this client's teaching? (Select all that apply.) a. "Take a 20-minute walk at least 5 days each week." b. "Attend local Alcoholics Anonymous (AA) meetings weekly." c. "Choose whole grains rather than foods with simple sugars." d. "Use cooking spray when you cook rather than margarine or butter." e. "Stay away from milk and dairy products that contain lactose." f. "We can talk to your doctor about a prescription for nicotine patches."

ANS: A Treatment of hepatitis C with ribavirin takes up to 48 weeks, making compliance a serious issue. The nurse should work with the client on a strategy to remain compliant for this length of time. Muscle aching is not a common side effect. The client will be on this medication for many weeks and does not need a blood toxicity examination. There is no need for the client to assess his or her radial pulse prior to taking the medication.

A nurse teaches a client with hepatitis C who is prescribed ribavirin (Copegus). Which statement should the nurse include in this client's discharge education? a. "Use a pill organizer to ensure you take this medication as prescribed." b. "Transient muscle aching is a common side effect of this medication." c. "Follow up with your provider in 1 week to test your blood for toxicity." d. "Take your radial pulse for 1 minute prior to taking this medication."

ANS: A, C, E - Rotavirus is more common among the youngest of clients. Not drinking water while swimming can help prevent E. coli infection. Parasitic diseases may take up to 2 weeks to become symptomatic. People with botulism need to be hospitalized to monitor for respiratory failure and paralysis. Escherichia coli is not transmitted by contact with infected animals.

A nurse teaches a community group about food poisoning and gastroenteritis. Which statements should the nurse include in this groups teaching? (Select all that apply.) a. Rotavirus is more common among infants and younger children. b. Escherichia coli diarrhea is transmitted by contact with infected animals. c. To prevent E. coli infection, dont drink water when swimming. d. Clients who have botulism should be quarantined within their home. e. Parasitic diseases may not show up for 1 to 2 weeks after infection.

Troponin level

A patient in the ED with chest pain has a possible myocardial infarction. Which lab result will determine this diagnosis?

Give the patient a back rub. A back rub reduces anxiety and can be delegated to the UAP. Once teaching has been done, the UAP can remind the patient to turn, but this is not related to relieving anxiety. Assessing anxiety and teaching are not within the scope of practice for the UAP.

A patient waiting for surgery is very anxious. What intervention can the nurse delegate to the unlicensed assistive personnel (UAP)?

Administer IV calcium gluconate. The patient's clinical manifestations of stridor and cramping are consistent with tetany caused by hypocalcemia resulting from damage to the parathyroid glands during surgery. Endotracheal intubation or tracheostomy may be needed if the calcium does not resolve the stridor. Suctioning will not correct the stridor.

A patient who had a subtotal thyroidectomy earlier today develops laryngeal stridor and a cramp in the right hand upon returning to the surgical nursing unit. Which collaborative action will the nurse anticipate next?

Bleeding Bronchospasm Mucosal damage

A patient with a tracheostomy who receives unnecessary suctioning can experience which complication? (Select all that apply)

Reduces obstruction of airways by decreasing inflammation

A patient with asthma has been prescribed a fluticasone (corticosteroids) inhaler. What is the purpose of this drug for this patient?

Protuberant A protuberant abdomen is rounded, bulging, and stretched (see Figure 21-7). A scaphoid abdomen caves inward.

A patient's abdomen is bulging and stretched in appearance. The nurse should describe this finding as:

Take a an apical pulse for 1 minute, noting any irregularity

A patient's bilateral radial pulses are occasionally weak and irregular. Which assessment technique does the nurse use first to investigate this finding?

ANS: B - The nurse should provide both approval and room for improvement in feedback after a teaching session. Feedback should be objective and constructive, and not evaluative. Reassuring the client that things will improve does not offer anything concrete for the client to work on, nor does it let him or her know what was done well. The nurse should not make the client convey learning needs because the client may not know what else he or she needs to understand. The client needs to become the expert in self-management of the ostomy, and the nurse should not offer to teach the daughter instead of the client.

After teaching a client who has a new colostomy, the nurse provides feedback based on the clients ability to complete self-care activities. Which statement should the nurse include in this feedback? a. I realize that you had a tough time today, but it will get easier with practice. b. You cleaned the stoma well. Now you need to practice putting on the appliance. c. You seem to understand what I taught you today. What else can I help you with? d. You seem uncomfortable. Do you want your daughter to care for your ostomy?

ANS: B, C, D - Parasitic infections can be transmitted to other people. The client himself or herself should keep the toilet area clean instead of possibly exposing another person to the disease. Parasites are transmitted via unclean water sources and sexual practices with rectal contact. The client should test his or her well water and ask sexual partners to have their stool examined for parasites. Raw vegetables are not associated with parasitic gastrointestinal infections. The client can eat vegetables from the store or a home garden as long as the water source is clean.

After teaching a client with a parasitic gastrointestinal infection, a nurse assesses the clients understanding. Which statements made by the client indicate that the client correctly understands the teaching? (Select all that apply.) a. Ill have my housekeeper keep my toilet clean. b. I must take a shower or bathe every day. c. I should have my well water tested. d. I will ask my sexual partner to have a stool test. e. I must only eat raw vegetables from my own garden.

ANS: D - Clients who have diverticular disease are prescribed a low-residue diet. Whole grains (rice pilaf), uncooked fruits and vegetables (salad, fresh fruit cup), and high-fiber foods (cup of bean soup) should be avoided with a low-residue diet. Canned or cooked vegetables are appropriate. Apple juice does not contain fiber and is acceptable for a low-residue diet.

After teaching a client with diverticular disease, a nurse assesses the clients understanding. Which menu selection made by the client indicates the client correctly understood the teaching? a. Roasted chicken with rice pilaf and a cup of coffee with cream b. Spaghetti with meat sauce, a fresh fruit cup, and hot tea c. Garden salad with a cup of bean soup and a glass of low-fat milk d. Baked fish with steamed carrots and a glass of apple juice

ANS: B - Toilet paper can irritate the sensitive perineal skin, so warm water rinses or soft cotton washcloths should be used instead. Although aloe vera may facilitate healing of superficial abrasions, it is not an effective skin barrier for diarrhea. Skin barriers such as zinc oxide and vitamin A and D ointment help protect the rectal area from the excoriating effects of liquid stools. Patting the skin is recommended instead of rubbing the skin dry.

After teaching a client with perineal excoriation caused by diarrhea from acute gastroenteritis, a nurse assesses the clients understanding. Which statement by the client indicates a need for additional teaching? a. Ill rinse my rectal area with warm water after each stool and apply zinc oxide ointment. b. I will clean my rectal area thoroughly with toilet paper after each stool and then apply aloe vera gel. c. I must take a sitz bath three times a day and then pat my rectal area gently but thoroughly to make sure I am dry. d. I shall clean my rectal area with a soft cotton washcloth and then apply vitamin A and D ointment.

A 58-year-old male who describes his pain as intense stabbing that spreads across his chest All patients who have chest pain would be assessed more thoroughly. To determine which patient should be seen first, the nurse must understand common differences in pain descriptions. Intense stabbing and viselike substernal pain that spreads through the patient's chest, arms, jaw, back, or neck are indicatives of a myocardial infarction. The nurse would plan to see this patient first to prevent cardiac cell death. A dull ache with numbness in the fingers is consistent with anxiety. Pain that gets worse with inspiration is usually related to a pleuropulmonary problem. Pain that spreads to the abdomen is often associated with an esophageal-gastric problem, especially when this pain is experienced by a male patient. Female patients may experience abdominal discomfort with a myocardial event. Although patients with anxiety, pleuropulmonary, and esophageal-gastric problems should be seen, they are not a higher priority than myocardial infarction.

An emergency department nurse triages patients who present with chest discomfort. Which patient would the nurse plan to assess first?

A, C, D, E, F Salmonella is usually contracted via contaminated eggs, beef, poultry, and green leafy vegetables. It is not transmitted through water in garden hoses or pools. Clients should wash leafy vegetables well, wash hands before and after using the restroom, make sure eggs and meat are cooked properly, and, because it can be transmitted by flies, keep flies off of food.

The client asks the nurse how to avoid becoming ill with Salmonella infection again. Which are appropriate responses from the nurse? (Select all that apply.) a. "Wash leafy vegetables carefully before eating or cooking them." b. "Do not ingest water from the garden hose or the pool." c. "Wash your hands before and after using the bathroom." d. "Be sure meat is cooked to the proper temperature." e. "Avoid eating eggs that are sunny side up or undercooked." f. "When eating outdoors, be sure to keep flies off your food."

Schedule of the client's follow-up examinations and x-ray assessments Correct: Because recurrence of gastric cancer is common, it will be a priority for the client to have follow-up examinations and x-rays, so that a recurrence can be detected quickly.

The client has been discharged home after surgery for gastric cancer, and a case manager will follow up with the client. To ensure a smooth transition from the hospital to the home setting, which information provided by the hospital nurse to the case manager is given the highest priority?

"She may have to be treated with chemotherapy drugs." Correct: The client may undergo treatment with chemotherapeutic agents to reduce the tumor before further treatment can commence.

The client is an older woman diagnosed with Zollinger-Ellison syndrome. Which statement made by the client's family demonstrates correct understanding of the disorder?

Treatment with radiation therapy Correct: Treatment with radiation therapy is known to be associated with the development of chronic gastritis.

The client is exhibiting symptoms of gastritis. The nurse is assessing the client to determine whether the form of gastritis being experienced is acute or chronic. Which data are correlated with a diagnosis of chronic gastritis?

Providing both oral and written instructions on changing the dressing and on symptoms of infection that must be reported to the physician Correct: Providing the spouse with both oral and written instructions on symptoms to report to the physician, as well as on how to perform the dressing change, will reinforce important points and boost the spouse's confidence.

The client is scheduled to be discharged after a gastrectomy. The client's spouse expresses concern that the client will be unable to change the surgical dressing adequately. What is the nurse's highest priority intervention?

Ask the client if a change in flavor would make the supplement more palatable. Correct: This action shows that the nurse is attempting to determine why the client is not drinking the supplements. Many clients cannot tolerate certain supplement flavors.

The client with gastric cancer is scheduled to undergo surgery to remove the tumor once 5 lbs of body weight has been regained. The client is not drinking the vanilla-flavored enteral supplements that have been prescribed. Which is the highest priority nursing intervention for this client?

ANS: A In obstructive jaundice, the normal flow of bile into the duodenum is blocked, allowing excess bile salts to accumulate on the skin. This leads to itching, or pruritus. The other statements are not accurate.

The client with obstructive jaundice asks the nurse why his skin is so itchy. Which is the nurse's best response? a. "Bile salts accumulate in the skin and cause the itching." b. "Toxins released from an inflamed gallbladder lead to itching." c. "Itching is caused by the release of calcium into the skin." d. "Itching is caused by a hypersensitivity reaction."

A Right lower quadrant pain, specifically at McBurney's point, is characteristic of appendicitis. Usually if nausea and vomiting begin first, the client has a gastroenteritis. Abdominal pain due to appendicitis decreases with knee flexion. Marked peristalsis and hyperactive bowel sounds are not indicative of appendicitis.

The nurse conducts a physical assessment for a client with abdominal pain. Which finding leads the nurse to suspect appendicitis? a. Severe, steady right lower quadrant (RLQ) pain b. Abdominal pain that started a day after vomiting began c. Abdominal pain that increases with knee flexion d. Marked peristalsis and hyperactive bowel sounds

Administering intravenous (IV) fluids Correct: Administration of IV fluids is necessary to treat the hypovolemia caused by acute GI bleeding.

The nurse finds a client vomiting coffee ground-type material. On assessment, the client has blood pressure of 100/74 mm Hg, is acutely confused, and has a weak and thready pulse. Which intervention will be the nurse's first priority?

2. A relaxed environment is an essential component of ulcer healing. Nurses can help clients understand the importance of relaxation and explore with them ways to balance work and family demands to promote healing. Being involved with his work may prevent boredom; however, this client is upset and argumentative. Not keeping up with his job will probably increase the client's stress level, but the nurse's response is best if it is based on the fact that a relaxed environment is an essential component of ulcer healing. Nurses cannot set limits on a client's behavior; clients must make the decision to make lifestyle changes.

The nurse finds a client who has been diagnosed with a peptic ulcer surrounded by papers from his briefcase and arguing on the telephone with a coworker. The nurse's response to observing these actions should be based on knowledge that: 1. Involvement with his job will keep the client from becoming bored. 2. A relaxed environment will promote ulcer healing. 3. Not keeping up with his job will increase the client's stress level. 4. Setting limits on the client's behavior is an important nursing responsibility.

3, 4, 2, 1 The client with peptic ulcer disease who is experiencing a sudden onset of acute stomach pain should be assessed first by the nurse. The sudden onset of stomach pain could be indicative of a perforated ulcer, which would require immediate medical attention. It is also important for the nurse to thoroughly assess the nature of the client's pain. The client with the fractured jaw is experiencing pain and should be assessed next. The nurse should then assess the client who is NPO for tests to ensure NPO status and comfort. Last, the nurse can assess the client before surgery.

The nurse has been assigned to provide care for four clients at the beginning of the day shift. In what order should the nurse assess these clients? 1. The client awaiting hiatal hernia repair at 11 am. 2. A client with suspected gastric cancer who is on nothing-by-mouth (NPO) status for tests. 3. A client with peptic ulcer disease experiencing sudden onset of acute stomach pain. 4. A client who is requesting pain medication 2 days after surgery to repair a fractured jaw.

B The nurse should provide both approval and room for improvement in feedback after a teaching session. Feedback should be objective and constructive, and not evaluative. Reassuring the client that things will improve does not offer anything concrete for the client to work on, nor does it let him or her know what was done well. The nurse should not make the client convey learning needs because the client may not know what else he or she needs to understand. The client needs to become the expert in self-management of the ostomy, and the nurse should not offer to teach the daughter instead of the client.

The nurse has completed the teaching session for a client with a new colostomy. Which feedback statement by the nurse is the most appropriate? a. "I realize that you had a tough time today, but it will get easier with practice." b. "You cleaned the stoma well. Now you need to practice putting on the appliance." c. "You seem to understand what I taught you today. What else can I help you with?" d. "You seem uncomfortable. Do you want your daughter to care for your ostomy?"

"Saline goes down the tube to help clean out your stomach." Correct: Gastric lavage involves the instillation of water or saline through an NG tube to clear out stomach contents.

The nurse has placed a nasogastric (NG) tube in the client who has overdosed to administer gastric lavage. The client asks the nurse about the purpose of the NG tube for the procedure. What is the nurse's best response?

B The client should not use enemas to promote elimination, but rather should rely on bulk-producing agents such as psyllium hydrophilic mucilloid (Metamucil). The other actions are appropriate.

The nurse has taught self-care measures to a client with an anal fissure. Which action by the client requires the nurse to do additional teaching? a. Taking warm sitz baths several times daily b. Administering daily enemas to prevent constipation c. Using bulk-producing agents to aid elimination d. Self-administering anti-inflammatory suppositories

pallor

The nurse in the PACU is assessing a post-op patient. Which of the following indicators suggest to the nurse as alteration in tissue perfusion?

Temperature Irritability Lethargy

The nurse in the neonatal intensive care unit is caring for an infant with myelomeningocele scheduled for surgical repair in the morning. Which early signs of infection should the nurse monitor on this infant (Select all that apply)?

fluid volume excess

The nurse is assessing a patient in the ER and notes the pulse to be bounding, neck veins distended, pitting edema in the lower extremities and crackles in the lung fields. what does the nurse suspect the patient to be displaying?

The patient cannot move the left arm and leg when asked to do so. The patient's inability to move the left arm and leg indicates that a hemorrhagic stroke may be occurring and will require immediate action to prevent further neurologic damage. The other clinical manifestations are also likely caused by the hypertension and will require rapid nursing actions, but they do not require action as urgently as the neurologic changes.

The nurse is assessing a patient who has been admitted to the intensive care unit (ICU) with a hypertensive emergency. Which finding is most important to report to the health care provider?

Perfusion

The nurse is assessing a patient with cardiovascular disease (CVD). What is the priority medical-surgical concept for this patient?

ANS: B, C, D, E Myocardial infarction (chest pain), pulmonary embolism (shortness of breath), adynamic ileus (lack of bowel sounds or flatus), and renal failure (urine output of 20 mL/6 hr) are just some of the complications that the nurse must monitor the client for after the Whipple procedure. Urinary retention is not a complication of this operation.

The nurse is caring for a client after a Whipple procedure. Which manifestations might indicate that a complication from the operation has occurred? (Select all that apply.) a. Urinary retention b. Substernal chest pain c. Shortness of breath d. Lack of bowel sounds or flatus e. Urine output of 20 mL/6 hr

ANS: C A transhepatic biliary catheter (T-tube) decompresses extrahepatic ducts to promote the flow of bile. When bile flows normally, it reaches the large intestine, where bile is converted to urobilinogen, coloring the stools brown. The other findings would not indicate successful T-tube placement.

The nurse is caring for a client who had a T-tube placed 3 days ago. Which assessment finding indicates to the nurse that the procedure was successful? a. Sclera that is slightly icteric b. Positive Blumberg's sign c. Soft, brown, formed stool this morning d. Sips of clear liquid tolerated without nausea

ANS: C Edema and low urine output following the Whipple procedure most likely are caused by hypoalbuminemia. Low albumin leads to third spacing of fluids and decreased intravascular fluids. As a result, edema and low urine output develop. Adding a colloid solution to the client's IV regimen will help shift edematous fluid from the interstitial space back into the intravascular space. Increasing the client's IV infusion rate will worsen the edema unless additional protein is added. Blood glucose monitoring and NG tubes are not related to this problem.

The nurse is caring for a client who had undergone a Whipple procedure 2 days previously. The nurse notes that the client's hands and feet are edematous, and urine output has decreased from the previous day. Which intervention does the nurse expect to provide for the client? a. Increase the client's IV fluid infusion rate. b. Monitor the client's blood sugar level every 4 hours. c. Add colloids to the client's IV solutions. d. Reinsert the client's nasogastric (NG) tube.

A Small bowel obstructions often lead to severe fluid and electrolyte imbalances. The client is hypokalemic (normal range, 3.5 to 5.0 mEq/L) and hyponatremic (normal range, 136 to 145 mEq/L). Dramatic weight loss without dieting followed by bowel obstruction leads to the probable development of colon cancer. High-pitched, hyperactive bowel sounds may be noted with large and small bowel obstructions. Crampy abdominal pain across the lower quadrants is associated with large bowel obstruction.

The nurse is caring for a client who has been diagnosed with a bowel obstruction. Which assessment finding leads the nurse to conclude that the obstruction is in the small bowel? a. Potassium of 2.8 mEq/L, with a sodium value of 121 mEq/L b. Losing 15 pounds over the last month without dieting c. Reports of crampy abdominal pain across the lower quadrants d. High-pitched, hyperactive bowel sounds in all quadrants

D Severe infection with Clostridium botulinum can lead to respiratory failure, so assessments of oxygen saturation and respiratory rate are of high priority for clients with suspected Clostridium botulinum infection. The other assessments may be completed after the respiratory system has been assessed.

The nurse is caring for a client who has food poisoning that may be the result of Clostridium botulinum infection. Which is the priority nursing assessment for this client? a. Heart rate and rhythm b. Bowel sounds and heart tones c. Fluid balance and urine output d. Oxygen saturation and respiratory rate

2, 4, 5. Following a gastroscopy, the nurse should monitor the client for complications, which include perforation and the potential for aspiration. An elevated temperature, complaints of epigastric pain, or the vomiting of blood (hematemesis) are all indications of a possible perforation and should be reported promptly. A sore throat is a common occurrence following a gastroscopy. Clients are usually sedated to decrease anxiety and the nurse would anticipate that the client will be drowsy following the procedure.

The nurse is caring for a client who has had a gastroscopy. Which of the following signs and symptoms may indicate that the client is developing a complication related to the procedure? Select all that apply. 1. The client has a sore throat. 2. The client has a temperature of 100 ° F (37.8 ° C). 3. The client appears drowsy following the procedure. 4. The client has epigastric pain. 5. The client experiences hematemesis.

ANS: C Before conducting an assessment about the client's feelings, the nurse should determine whether he or she is willing and able to talk about them. If the client is open to the conversation and his or her room is not appropriate, an alternative meeting space may be located. The nurse should be present for the client during this time, and pulling up a chair and sitting with the client indicates that presence. Because the nurse is assessing the client's response to a terminal diagnosis, it is not necessary to have detailed information about the projected prognosis; the nurse knows that the client is facing an end-of-life illness.

The nurse is caring for a client who has just been diagnosed with end-stage pancreatic cancer. The nurse assesses the client's emotional response to the diagnosis. Which is the nurse's initial action for the assessment? a. Bring the client to a quiet room for privacy. b. Pull up a chair and sit next to the client's bed. c. Determine whether the client feels like talking about his or her feelings. d. Review the health care provider's notes about the prognosis for the client.

1. A sudden spike in temperature following an endoscopic procedure may indicate perforation of the GI tract. The nurse should promptly conduct a further assessment of the client, looking for further indicators of perforation, such as a sudden onset of acute upper abdominal pain; a rigid, boardlike abdomen; and developing signs of shock. Telling the assistant to change thermometers is not an appropriate action and only further delays the appropriate action of assessing the client. The nurse would not administer acetaminophen without further assessment of the client or without a physician's order; a suspected perforation would require that the client be placed on nothing-by-mouth status. Asking the assistant to bathe the client before any assessment by the nurse is inappropriate.

The nurse is caring for a client who has just had an upper GI endoscopy. The client's vital signs must be taken every 30 minutes for 2 hours after the procedure. The nurse assigns an unlicensed nursing personnel (UAP) to take the vital signs. One hour later, the UAP reports the client, who was previously afebrile, has developed a temperature of 101.8 ° F (38.8 ° C). What should the nurse do in response to this reported assessment data? 1. Promptly assess the client for potential perforation. 2. Tell the assistant to change thermometers and retake the temperature. 3. Plan to give the client acetaminophen (Tylenol) to lower the temperature. 4. Ask the assistant to bathe the client with tepid water.

ANS: C Drainage from the JP drain initially appears serosanguineous in color. The drainage will appear bile-colored within 24 hours. The nurse does not need to notify the surgeon, milk the tubing, or irrigate the drain because this is an expected finding.

The nurse is caring for a client who has just undergone traditional cholecystectomy surgery and has a Jackson-Pratt (JP) drain in place. The nurse notes serosanguineous drainage present in the drain. Which is the nurse's priority action? a. Gently milk the drain tubing. b. Notify the surgeon immediately. c. Document the finding in the client's chart. d. Irrigate the drain with sterile normal saline.

ANS: B The nurse assesses the skin around the drainage tube for redness or skin irritation, which can be severe from leakage of pancreatic enzymes. The nurse applies a skin barrier such as Stomahesive around the drainage tube to prevent excoriation. A side-lying position may be more comfortable for the client. The drainage tube should not be clamped or irrigated without specific orders.

The nurse is caring for a client who has undergone surgery to drain a pancreatic pseudocyst with placement of a pancreatic drainage tube. Which nursing intervention prevents complications from this procedure? a. Positioning the client in a right side-lying position b. Applying a skin barrier around the drainage tube site c. Clamping the drainage tube for 2 hours every 12 hours d. Irrigating the drainage tube daily with 30 mL of sterile normal saline

ANS: B, D, F The client should be advised to stay sober, and AA is a great resource. The client requires a low-fat diet, and cooking spray is low in fat compared with butter or margarine. If the client smokes, he or she must stop because nicotine can precipitate an exacerbation. A nicotine patch may help the client quit smoking. The client must rest until his or her strength returns. The client requires high carbohydrates and calories for healing; complex carbohydrates are not preferred over simple ones. Dairy products do not cause a problem.

The nurse is caring for a client who is being discharged from the hospital after an attack of acute pancreatitis. Which discharge instructions does the nurse provide for the client to help prevent a recurrence? (Select all that apply.) a. "Take a 20-minute walk at least 5 days each week." b. "Attend local Alcoholics Anonymous (AA) meetings weekly." c. "Choose whole grains rather than foods with simple sugars." d. "Use cooking spray when you cook rather than margarine or butter." e. "Stay away from milk and dairy products that contain lactose." f. "We can talk to your doctor about a prescription for nicotine patches."

A Common side effects of 5-FU include fatigue, leukopenia, diarrhea, mucositis and mouth ulcers, and peripheral neuropathy. However, the client's WBC count is very low (normal range, 5000 to 10,000/mm3), so the provider should be notified. He or she may want to delay chemotherapy by a day or two. Certainly the client is at high risk for infection. The other assessment findings are consistent with common side effects of 5-FU that would not need to be reported immediately.

The nurse is caring for a client who is to receive 5-fluorouracil (5-FU) chemotherapy IV for the treatment of colon cancer. Which assessment finding leads the nurse to contact the health care provider? a. White blood cell (WBC) count of 1500/mm3 b. Presence of fatigue with a headache c. Presence of slight nausea and no appetite d. Two diarrhea stools yesterday

A The presence of strictures predisposes the client to intestinal obstruction. Abdominal distention may indicate that the client has developed an obstruction of the large bowel, and the client's provider should be notified right away. Low-grade fever, bloody diarrhea, and crampy abdominal pain are common symptoms of Crohn's disease.

The nurse is caring for a client with Crohn's disease and colonic strictures. Which assessment finding requires the nurse to consult the health care provider immediately? a. Distended abdomen b. Temperature of 100.0° F (37.8° C) c. Traces of blood in the stool d. Crampy lower abdominal pain

D The client with Crohn's disease is already at risk for malabsorption and malnutrition. Malnutrition impairs healing of the fistula and immune responses. Therefore, maintaining adequate nutrition is a priority for this client. The client will require 3000 calories per day to promote healing of the fistula. Monitoring the client's blood sugar and hemoglobin levels is important, but less so than encouraging nutritional intake. The client need not be positioned to facilitate gravity drainage of the fistula, because fistulas often are found in the abdominal cavity.

The nurse is caring for a client with Crohn's disease who has developed a fistula. Which nursing intervention is the highest priority? a. Monitor the client's hematocrit and hemoglobin. b. Position the client to allow gravity drainage of the fistula. c. Check and record blood glucose levels every 6 hours. d. Encourage the client to consume a diet high in protein and calories.

A Flagyl is the drug of choice for Giardia lamblia infection. Cipro and Rocephin are antibiotics used for bacterial infections. Azulfidine is used for ulcerative colitis and Crohn's disease.

The nurse is caring for a client with Giardia lamblia infection. Which medication does the nurse anticipate teaching the client about? a. Metronidazole (Flagyl) b. Ciprofloxacin (Cipro) c. Sulfasalazine (Azulfidine) d. Ceftriaxone (Rocephin)

A Parasitic infections can be transmitted to other people. The client himself or herself should keep the toilet area clean instead of possibly exposing another person to the disease. The other statements are accurate

The nurse is caring for a client with a parasitic gastrointestinal infection. What statement by the client indicates a need for further teaching? a. "I will have my housekeeper keep my toilet very clean." b. "I need to shower or bathe every day." c. "I need to have my well water tested." d. "My sexual partner needs to have a stool test."

ANS: D Grayish-blue discoloration on the flanks (Turner's sign) indicates pancreatic enzyme leakage into the peritoneal cavity. This presents a risk of shock for the client, so IV access should be maintained with at least one large-bore patent IV catheter. The client may or may not need surgery; usually a fetal position helps with pain, and having an NG tube would not take priority over IV access.

The nurse is caring for a client with acute pancreatitis. During the physical assessment, the nurse notes a grayish-blue discoloration of the client's flanks. Which is the nurse's priority action? a. Prepare the client for emergency surgery. b. Place the client in high Fowler's position. c. Insert a nasogastric (NG) tube to low intermittent suction. d. Ensure that the client has a patent large-bore IV site.

ANS: B The client should be kept NPO to reduce GI activity and reduce pancreatic enzyme production. IV fluids should be used to prevent dehydration. The client may need a nasogastric (NG) tube. Pain medications should be given around the clock and more frequently than every 4 to 6 hours. A fetal position with legs drawn up to the chest will promote comfort.

The nurse is caring for a client with acute pancreatitis. Which nursing intervention best reduces discomfort for the client? a. Administering morphine sulfate IV every 4 to 6 hours as needed b. Maintaining NPO status for the client with IV fluids c. Providing small, frequent feedings, with no concentrated sweets d. Placing the client in semi-Fowler's position at elevation of 30 degrees

ANS: C Jaundice, clay-colored stools, and dark urine are more commonly seen with chronic than with acute cholecystitis. The other symptoms are seen equally with both conditions.

The nurse is caring for a client with cholecystitis. The client is a poor historian and is unable to tell the nurse when the symptoms started. Which assessment finding indicates to the nurse that the condition is chronic rather than acute? a. Temperature of 100.1° F (37.8° C) b. Positive Murphy's sign c. Light-colored stools d. Upper abdominal pain after eating

ANS: C In chronic cholecystitis, bile duct obstruction results in the absence of urobilinogen to color the stool. Excess circulating bilirubin turns the urine dark and foamy. The other assessment findings do not correlate with chronic cholecystitis.

The nurse is caring for a client with cholecystitis. Which assessment finding indicates to the nurse that the condition is chronic rather than acute? a. Abdomen that is hyperresonant to percussion b. Hyperactive bowel sounds and diarrhea c. Clay-colored stools and dark amber urine d. Rebound tenderness in the right upper quadrant

ANS: B A client with chronic pancreatitis needs 4000 to 6000 calories per day for optimum nutrition and healing. The client may have additional restrictions if he or she has other health problems such as diabetes. The nurse should collaborate with the registered dietitian to help the client plan nutritional intake.

The nurse is caring for a client with chronic pancreatitis. Which instruction by the nurse is most appropriate? a. "You will need to limit your protein intake." b. "We need to call the dietitian to get help in planning your diet." c. "You cannot eat concentrated sweets any longer." d. "Try to eat less red meat and more chicken and fish."

B Nurses need to become familiar with community-based resources to assist clients better. The local chapter of the United Ostomy Associations of America has resources for clients and their families, including Ostomates (specially trained visitors who also have ostomies). Although the enterostomal therapist is an expert in ostomy care, talking with him or her is not the same as talking with someone who actually has had a colostomy. Many people are willing to share their ostomy experience in the hope of helping others. The nurse should not brush aside the client's request by saying that no colostomy clients are present on the unit at the time.

The nurse is caring for a client with colon cancer and a new colostomy. The client wishes to talk with someone who had a similar experience. Which is the nurse's best response? a. "Most people who have had a colostomy are reluctant to talk about it." b. "I will make a referral to the United Ostomy Associations of America." c. "You can get all the information you need from the enterostomal therapist." d. "I do not think that we have any other clients with colostomies on the unit right now."

ANS: A The client is not asking the nurse actually to explain why the cancer has occurred, but simply to validate that no easy or straightforward answer can be found.

The nurse is caring for a client with end-stage pancreatic cancer. The client asks the nurse, "Why is this happening to me?" Which is the nurse's best response? a. "I don't know. I wish I had an answer for you, but I don't." b. "It's important to keep a positive attitude for your family right now." c. "Scientists have not determined why cancer develops in certain people." d. "I think that this is a trial so you can become a better person because of it."

ANS: A, D, F Obesity, pregnancy, and diabetes are all risk factors for the development of cholelithiasis. Moderate alcohol intake and a diet low in saturated fats may decrease the risk. Metabolic syndrome is a precursor to diabetes, and the client should be informed of the connection.

The nurse is caring for a female client with cholelithiasis. Which assessment findings from the client's history and physical examination may have contributed to development of the condition? (Select all that apply.) a. Body mass index (BMI) of 46 b. Vegetarian diet c. Drinking 4 ounces of red wine nightly d. Pregnant with twins e. History of metabolic syndrome f. Glycosylated hemoglobin level of 15%

C The ostomy nurse is a valuable resource for clients, providing suggestions for supplies and methods to manage the ostomy. A larger dress size will not necessarily help hide the ostomy appliance. Avoiding broccoli and carbonated drinks does not offer reassurance for the client. Ileostomies have an almost constant liquid effluent, so pouch removal during the prom is not feasible.

The nurse is caring for a teenage girl with a new ileostomy. She tells the nurse tearfully that she cannot go to the prom with an ostomy. Which is the nurse's best response? a. "You should get your prom dress one size larger to hide the ostomy appliance." b. "You should avoid broccoli and carbonated drinks so that the pouch won't fill with air under your dress." c. "Let's talk to the enterostomal therapist (ET) about options for ostomy supplies and dress styles so that you can look beautiful for the prom." d. "You can remove the pouch from your ostomy appliance when you are at the prom so that it is less noticeable."

Pain occurs 1 1/2 to 3 hours after a meal, usually at night. Correct: A key symptom characteristic of duodenal ulcers is that pain usually awakens the client between 1 AM and 2 AM, occurring 1 1/2 to 3 hours after a meal.

The nurse is caring for an older adult male client who reports stomach pain and heartburn. Which syndrome is most significant in determining whether the client's ulceration is gastric or duodenal in origin?

B Dehydration can occur quickly in older clients with Salmonella food poisoning caused by diarrhea, so maintenance of fluid balance is a high priority. Monitoring vital signs and providing perineal care are important nursing actions, but are of lower priority than fluid replacement. Contact Isolation is not regularly instituted for Salmonella infection. Standard Precautions are usually sufficient.

The nurse is caring for an older client with Salmonella food poisoning. Which is the priority action of the nurse? a. Monitor vital signs. b. Maintain IV fluids. c. Provide perineal care. d. Initiate Isolation Precautions.

C Lomotil can cause drowsiness and can increase the older client's risk for falls. The nurse should consult with the provider to see if this medication is really necessary and, if an antidiarrheal medication is warranted, what other options might be available. The other orders are appropriate, although the nurse would have to monitor the client's total 24-hour Tylenol dosage to ensure that the client did not receive more than 4000 mg/24 hr.

The nurse is caring for an older client with gastroenteritis. Which order does the nurse consult with the health care provider about? a. IV 0.45% NS at 50 mL/hr b. Clear liquids as tolerated c. Diphenoxylate hydrochloride/atropine sulfate (Lomotil) orally, after each loose stool d. Acetaminophen (Tylenol), 325-650 mg orally every 4 hr PRN pain

A, D, F An 8 Fr nasogastric tube is too small for drainage of thick stomach contents. Sterile gloves are not needed for the procedure. The tube should be secured to the clients gown, not to the pillowcase, because it could become dislodged easily. The clients head should be flexed forward once the tube has reached the oropharynx. All the other actions are appropriate. A 60-mL irrigation syringe should be attached to the end of the tube before insertion so that gastric fluid does not erupt from the tube when it enters the stomach.

The nurse is helping a student prepare to insert a nasogastric tube for an adult client with a bowel obstruction. Which actions by the student indicate to the nurse that a review of the procedure is needed? (Select all that apply.) a. Gathering supplies, including an 8 Fr Levin tube, sterile gloves, tape, and water-soluble lubricant b. Performing hand hygiene and positioning the client in high Fowler's position, with pillows behind the head and shoulders c. Attaching a 60-mL irrigation syringe to the end of the nasogastric tube before inserting it into the nose d. Instructing the client to extend the neck against the pillow once the nasogastric tube has reached the oropharynx e. Checking for correct placement by checking the pH of the fluid aspirated from the tube f. Securing the nasogastric tube by taping it to the client's nose and pinning the end to the pillowcase g. Connecting the nasogastric tube to intermittent medium suction with an anti-reflux valve on the air vent

Blood pressure from 140/90 to 110/70 mm Hg Correct: A decrease in blood pressure is the most indicative sign of bleeding.

The nurse is monitoring the client with gastric cancer for signs and symptoms of upper GI bleeding. Which change in vital signs is most indicative of bleeding related to cancer?

B Assessment findings indicate that the client may have an overfull bladder. In the immediate postoperative period, the client may experience difficulty voiding owing to urinary retention. A rectal tube should not be inserted for a client who had a hemorrhoidectomy the previous day. The client's vital signs may be checked after the nurse determines the client's last void. The nurse should document all findings and actions in the client's medical record.

The nurse is performing a physical assessment for a client who underwent a hemorrhoidectomy the previous day. The nurse notes that the client has lower abdominal distention accompanied by dullness to percussion over the distended area. Which is the nurse's priority action? a. Assess the client's vital signs. b. Determine the last time the client voided. c. Insert a rectal tube to facilitate passage of flatus. d. Document the findings in the client's chart.

C The presence of visible peristaltic waves, accompanied by high-pitched or tingling bowel sounds, is indicative of partial obstruction caused by the tumor. Assessment findings do not indicate metastasis to the liver, intussusception of the intestine, or toxic megacolon.

The nurse is performing a physical assessment of a client with a new diagnosis of colorectal cancer. The nurse notes the presence of visible peristaltic waves and, on auscultation, hears high-pitched bowel sounds. Which conclusion does the nurse draw from these findings? a. The tumor has metastasized to the liver and biliary tract. b. The tumor has caused an intussusception of the intestine. c. The growing tumor has caused a partial bowel obstruction. d. The client has developed toxic megacolon from the growing tumor.

A This type of hernia is associated with obesity. The other assessment findings do not place the client at increased risk for an acquired umbilical hernia.

The nurse is performing a physical examination on a client. Which assessment finding leads the nurse to check the client's abdomen for the presence of an acquired umbilical hernia? a. Body mass index (BMI) of 41.9 b. Cholecystectomy last year c. History of irritable bowel syndrome d. Daily dose of lansoprazole (Prevacid) 30 mg orally

C The nurse should wait until the client is ready to look at the ostomy and stoma before initiating teaching about ostomy care. The nurse should monitor clues from the client and encourage him or her to start taking an active role in management. Effective learning will occur only when the learner is ready. The other considerations are of lower priority for the client and nurse.

The nurse is preparing to begin teaching the client about how to care for a new ileostomy. Which consideration is the highest priority for the nurse when planning teaching for this client? a. Informing the client about what to expect with basic ostomy care b. Starting the teaching after the client has received pain medication c. Starting the teaching when the client is ready to look at the stoma d. Making sure that all needed supplies are ready at the client's bedside

3. Diet therapy for ulcer disease is a controversial issue. There is no scientific evidence that diet therapy promotes healing. Most clients are instructed to follow a diet that they can tolerate. There is no need for the client to ingest only a bland or high-protein diet. Milk may be included in the diet, but it is not recommended in excessive amounts.

The nurse is preparing to teach a client with a peptic ulcer about the diet that should be followed after discharge. The nurse should explain that the diet will most likely consist of which of the following? 1. Bland foods. 2. High-protein foods. 3. Any foods that are tolerated. 4. Large amounts of milk.

B Cipro should be taken for 10 to 14 days to treat Salmonella infection, and should not be stopped once the diarrhea has cleared. Clients should be advised to take the entire course of medication. People with Salmonella should not prepare foods for others because the infection may be spread in this way. Dishes and eating utensils should not be shared and should be cleaned thoroughly. Hands should be washed with antibacterial soap before and after eating to prevent spread of the bacteria. Clients can be carriers for up to 1 year.

The nurse provides discharge teaching for a client who was hospitalized for Salmonella food poisoning. Which client statement indicates that additional teaching is needed? a. "I will let my husband do the cooking for my family." b. "I will take the ciprofloxacin (Cipro) until the diarrhea has resolved." c. "I will wash my hands with antibacterial soap before and after each meal." d. "I will make sure that my dishes go straight into the dishwasher after each meal."

B, C, G The client should avoid using soap to clean around the stoma because it might prevent effective adhesive of the ostomy appliance. The client should use warm water and a soft washcloth instead. The stoma should remain a soft pink color. A deep red or purple hue indicates ischemia and should be reported to the surgeon right away. The tissue of the stoma is very fragile, and scant bleeding may occur when the stoma is cleaned. Yogurt and buttermilk can help reduce gas in the pouch, so the client need not avoid dairy products. Exercise (other than some contact sports) is important for clients with an ostomy.

The nurse is providing discharge teaching for a client who has undergone colon resection surgery with a colostomy. Which statements by the client indicate that the instruction was understood? (Select all that apply.) a. "I will change the ostomy appliance daily and as needed." b. "I will use warm water and a soft washcloth to clean around the stoma." c. "I will start bicycling and swimming again once my incision has healed." d. "I will notify the doctor right away if any bleeding from the stoma occurs." e. "I will check the stoma regularly to make sure that it stays a deep red color." f. "I will avoid dairy products to reduce gas and odor in the pouch." g. "I will cut the flange so it fits snugly around the stoma to avoid skin breakdown."

B The dressing should be changed every day until the staples are removed, so the client can check the incision for signs of infection. Constipation is common following hernia surgery, so clients should include adequate amounts of fiber in the diet. The maximum daily dosage of Tylenol is 4000 mg. Taking 1000 mg of Tylenol every 4 hours means that intake is 6000 mg/day, which could cause toxicity and liver damage. The client should change positions and take deep breaths to facilitate lung expansion but should avoid coughing, which can place stress on the incision line.

The nurse is providing preoperative teaching for a client who will undergo herniorrhaphy surgery. Which instruction does the nurse give to the client? a. "Eat a low-residue diet for the first week after surgery." b. "Change the dressing every day until the staples are removed." c. "Take acetaminophen (Tylenol) 1000 mg every 4 hours for pain." d. "Cough and deep breathe every 2 hours for the first week after surgery."

Place nasogastric (NG) tube, and connect to suction. Correct: To decrease spillage of duodenal contents into the peritoneum, NG suction should be rapidly initiated. This will minimize the risk for peritonitis.

The nurse is reviewing admitting requests for a client admitted to the intensive care unit with perforation of a duodenal ulcer. Which request will the nurse implement first?

D A client with an ileostomy should call the provider if no drainage has come from the ostomy in 6 to 12 hours. The other statements indicate good understanding of self-management.

The nurse is teaching a client how to care for a new ileostomy. Which client statement indicates that additional teaching is needed? a. "I will consult the pharmacist before filling any new prescriptions." b. "I will empty the ostomy pouch when it is half-filled with stool or gas." c. "I will wash my hands with antibacterial soap before and after ostomy care." d. "I will call my health care provider if I have not had ostomy drainage for 3 hours."

A The client is instructed to apply the truss before arising, not before going to bed at night. The other statements show accurate knowledge in using a truss.

The nurse is teaching a client how to use a truss for a femoral hernia. Which statement by the client indicates the need for further teaching? a. "I will put on the truss before I go to bed each night." b. "I will put some powder under the truss to avoid skin irritation." c. "The truss will help my hernia because I can't have surgery." d. "If I have abdominal pain, I will let my health care provider know right away."

D The client should be instructed to use wet wipes and dab the anal area after defecating to avoid further irritation. Dibucaine can be used only for short periods of time because long-term use can mask worsening symptoms. Clients with hemorrhoids require high-fiber foods. The client should not be encouraged to strain at stool or to spend long periods of time on the toilet, because this increases pressure in the rectal area, which can make hemorrhoids worse.

The nurse is teaching self-care measures for a client who has hemorrhoids. Which nursing intervention does the nurse include in the plan of care for the client? a. Instruct the client to use dibucaine (Nupercainal) ointment whenever needed. b. Teach the client to choose low-fiber foods to make bowels move more easily. c. Tell the client to take his or her time on the toilet when needing to defecate. d. Encourage the client to dab with moist wipes instead of wiping with toilet paper.

"Ice cream can be eaten in moderation." Correct: Milk products such as ice cream must be eliminated from the diet of the client with dumping syndrome.

The nurse is teaching the client about dietary choices to prevent dumping syndrome after gastric bypass surgery. Which statement by the client indicates a need for further teaching?

"Small meals should be eaten about six times a day." Correct: The client with chronic gastritis should eat six small meals daily to avoid symptoms.

The nurse is teaching the client how to prevent recurrent chronic gastritis symptoms before discharge. Which statement by the client demonstrates correct understanding of the nurse's instruction?

"Nizatidine (Axid) needs to be taken three times a day to be effective." Correct: Nizatidine (Axid) is most effective if administered twice daily.

The nurse is teaching the client with peptic ulcer disease (PUD) about the prescribed drug regimen. Which statement made by the client indicates a need for further teaching before discharge?

A In a reducible hernia, the contents of the hernial sac can be replaced into the abdominal cavity by gentle pressure or by lying flat. The contents of irreducible, strangulated, or incarcerated hernias may not be replaced into the abdomen when the client lies down.

The nurse notes a bulge in a client's groin that is present when the client stands and disappears when the client lies down. Which conclusion does the nurse draw from these assessment findings? a. Reducible inguinal hernia b. Indirect umbilical hernia c. Strangulated ventral hernia d. Incarcerated femoral hernia

ACE inhibitors Beta blockers Angiotensin II receptor blockers

The nurse prepares to teach a patient recovering from a MI about combination drug therapy for controlling hypertension. Which drugs does the nurse include in the teaching plan?(Select all tha apply)

A Long-term inflammatory bowel disease increases the risk of colon cancer, so regular colonoscopies are recommended. A high-fiber diet is not recommended for clients with Crohn's disease because fiber can further irritate the inner lining of the bowel. Asacol (mesalamine [5-aminosalicylic acid]) should be taken daily, not as needed. Avoiding heavy lifting and tight-fitting clothes is not necessary.

The nurse reviews a health teaching for a client with Crohn's disease. Which instruction does the nurse provide for the client? a. "You should have a colonoscopy every few years." b. "You should eat a diet that is high in protein and fiber." c. "You should avoid heavy lifting and tight-fitting clothes." d. "You should take the Asacol whenever you have loose stools."

Misoprostol (Cytotec) Correct: Misoprostol (Cytotec) is a prostaglandin analogue that protects against NSAID-induced ulcers.

The nurse reviews a medication history for a client newly diagnosed with peptic ulcer disease (PUD) who has a history of using ibuprofen (Advil, Motrin, others) frequently for chronic knee pain. The nurse anticipates that the health care provider will request which medication for this client?

Patient with a spinal cord injury who cannot tolerate sitting up Patients on bisphosphonates must be able to sit upright for 30 to 60 minutes after taking them. The patient who cannot tolerate sitting up is not a good candidate for this class of drug. Poor renal function also makes patients bad candidates for this drug, but the patient with a creatinine of 0.8 mg/dL (61 mcmol/L) is within normal range. Diabetes and hypertension are not related unless the patient also has renal disease. The patient who recently fell and sustained fractures is a good candidate for this drug if the fractures are related to osteoporosis.

The nurse sees several patients with osteoporosis. For which patient would bisphosphonates not be a good option?

Young adult with epigastric pain, hiccups, and abdominal distention after having a total gastrectomy Correct: This client is experiencing symptoms of acute gastric dilation, which can disrupt the suture line. The surgeon should be notified immediately because the nasogastric tube may need irrigation or repositioning.

The nurse working during the day shift on the medical unit has just received report. Which client will the nurse plan to assess first?

Oatmeal The child with celiac disease is unable to fully digest gluten, the protein found in wheat, barley, rye, and oats. Oatmeal contains gluten and is not an appropriate food selection. Rice is an appropriate choice because it does not contain gluten. Corn is digestible because it does not contain gluten. Meats do not contain gluten and can be included in the diet of a child with celiac disease.

What food choice by the parent of a 2-year-old child with celiac disease indicates a need for further teaching?

Tissue integrity

What is the most important interrelated medical-surgical concept for nursing care of patients requiring oxygen therapy?

Exceed the typical healing time, but acute wounds heal readily The length of time for healing is the determining factor when classifying a wound as acute or chronic. Acute wounds are expected to be of short duration. Wounds that exceed the anticipated length of recovery are classified as chronic wounds.

What is the primary difference between acute and chronic wounds? Chronic wounds:

Corticosteroids Corticosteroids such as prednisone and prednisolone are used in short bursts to suppress the inflammatory response in inflammatory bowel disease. Antacids and antidiarrheals are not drugs of choice to treat the inflammatory process of inflammatory bowel disease. Antibiotics may be used as adjunctive therapy to treat complications.

What is used to treat moderate-to-severe inflammatory bowel disease?

Sudden relief from pain Signs of peritonitis, in addition to fever, include sudden relief from pain after perforation. Tachycardia, not bradycardia, is a manifestation of peritonitis. Anorexia is already a clinical manifestation of appendicitis. Abdominal distention usually increases in addition to an increase in pain (usually diffuse and accompanied by rigid guarding of the abdomen).

When caring for a child with probable appendicitis, the nurse should be alert to recognize what sign of perforation?

Uncontrolled hypertension All of the factors contribute to the patient's risk, but only hypertension can potentially be modified to decrease the patient's risk for further expansion of the aneurysm.

When discussing risk factor modification for a 63-year-old patient who has a 5-cm abdominal aortic aneurysm, the nurse will focus discharge teaching on which patient risk factor?

3, 4, 5. Vomiting and weight loss are common with gastric ulcers. The client may also have blood in the stools (melena) from gastric bleeding. Clients with a gastric ulcer are most likely to complain of a burning epigastric pain that occurs about 1 hour after eating. Eating frequently aggravates the pain. Clients with duodenal ulcers are more likely to complain about pain that occurs during the night and is frequently relieved by eating.

When obtaining a nursing history on a client with a suspected gastric ulcer, which signs and symptoms should the nurse expect to assess? Select all that apply. 1. Epigastric pain at night. 2. Relief of epigastric pain after eating. 3. Vomiting. 4. Weight loss. 5. Melena.

take frequent rest periods while performing activities and use purse-lip breathing

When the home care nurse notes that a moderately obese male client with COPD becomes short of breath while performing self care activities. The nurse encourages the client to:

Auscultate the patient's breath sounds. The initial action should be to assess the patient further because the history and symptoms are consistent with several possible complications of central line insertion, including embolism and pneumothorax. The other actions may be appropriate, but further assessment of the patient is needed before notifying the health care provider, offering reassurance, or administration of morphine.

Which action should the nurse take first when a patient complains of acute chest pain and dyspnea soon after insertion of a centrally inserted IV catheter?

Decreased cardiac output Any abnormal electrical activity of the heart causes decreased cardiac output.

Which client problem has priority for the clinet with a cardiac dysrhythmia?

Venturi mask

Which high flow oxygen delivery system delivers the most accurate concentration of oxygen without intubation?


Set pelajaran terkait

deca Business Management/Admin prep cluster Exam (12-13) #2

View Set

Evolution of European Thought Take-home Test

View Set

OOP + Python3 Basics coding exercises

View Set